Sie sind auf Seite 1von 39

MAT7400 Assignment #3

THIS PAGE IS INTENTIONALLY BLANK

MAT7400 Assignment #3

Dummit & Foote Text Exercise 3.4-2: Exhibit all 3 composition series for Q8 and all 7 composition
series for D8 . List the composition factors in each case.
Solution: In Exercise 3.1-32, we showed every subgroup of Q8 is normal. Of the normal subgroups, hii,
hji, and hki are maximal. In addition, these three only have one non-trivial subgroup. Every quotient in
each series has order two:
Q8 /hii, hii/h1i/, h1i/1,
Q8 /hji, hji/h1i/, h1i/1,
Q8 /hki, hki/h1i/, h1i/1,
So the every quotient is isomorphic to the simple abelian group Z/2Z. This gives us the following three
composition series for Q8 :
1 C h1i C hii C Q8 ,
1 C h1i C hji C Q8 ,
1 C h1i C hki C Q8 .
Exercise 3.1-33 informed us that there are three maximal subgroups of D8 which are normal: hs, r2 i, hri,
and hrs, r2 i. Each have order four.
hs, r2 i has three subgroups of order two: hsi, hr2 i, hr2 si. Since each of these subgroups have index two,
they are normal to hs, r2 i.
hri has one subgroup of order two: hr2 i and since its index is two, it is normal in hri.
hrs, r2 i has three subgroups of index two (and hence normal): hr2 i, hrsi, hr3 si.
Again, every quotient is isomorphic to the simple abelian group Z/2Z. This gives us the following seven
composition series for D8 :
1. 1 C hsi C hs, r2 i C D8
2. 1 C hr2 si C hs, r2 i C D8
3. 1 C hr2 i C hs, r2 i C D8
4. 1 C hr2 i C hri C D8
5. 1 C hr2 i C hrs, r2 i C D8
6. 1 C hrsi C hrs, r2 i C D8
7. 1 C hr3 si C hrs, r2 i C D8
Dummit & Foote Text Exercise 3.4-5: Prove that subgroups and quotient groups of a solvable group
are solvable.
Solution: A subnormal series of a group G is a chain of subgroups
G = Gs > G2 > G1 > G0
such that Gi E Gi+1 for all i.
Let G be a solvable group. So G has a subnormal series 1 = G0 E G1 E E Gs1 E Gs = G with abelian
factors: for each i = 1, . . . , s, Gi+1 /Gi is an abelian group. Let H G. Consider H Gi and H Gi+1 .
Each is a group. If g H Gi and h H Gi+1 , then hgh1 H since g, h H. Also, hgh1 Gi . Since
Gi E Gi+1 , H Gi C H Gi+1 . This also implies that H Gi+1 NG (H Gi ). Hence H is solvable and
the composition (subnormal) series is
1 E H G1 E E H Gs1 E H Gs = H G = H.
We can also use the second (diamond) isomorphism theorem to show this as well. Again let H G and for
each i let Hi = Gi H. Then for every i, Gi1 E Gi , and we have the subnormal series
1 = H0 E H1 E E Hs1 E Hs = H.
By the second (diamond) isomorphism theorem, for every i,
Hi /Hi1 = Hi /(Gi1 H)
= (Hi Gi1 )/Gi1 ,

MAT7400 Assignment #3

which is a subgroup of the abelian group Gi /Gi1 and therefore is abelian. Hence, H has a subnormal
series with abelian factors and that implies H is solvable.
(I had help with this part of problem.)
Let K be a quotient group of G with K = G/N with N E G. Define : G K as the projection
homomorphism. and last define Ki = (Hi ). Then for each i, Ki1 E Ki . We have now obtained the
subnormal series:
1 = K0 E K1 E E Ks1 E Ks = K.
Again by the second (diamond) isomorphism theorem we obtain
Ki /Ki1 = (Gi N )/(Gi1 N )
= Gi /(Gi Gi1 N )
which is a quotient of Gi /Gi1 by the third isomorphism theorem since Gi1 E Gi Gi1 N . Thus, the
quotient groups Ki /Ki1 are abelian and therefore K is solvable.//
Dummit & Foote Text Exercise 3.5-10: Find a composition series for A4 . Deduce that A4 is solvable.
Solution: It is quite fortuitous for me as Prof. Okoh had a very similar problem assigned as an exam
problem!
Let H = {(), (1, 2)(3, 4), (1, 3)(2, 4), (1, 4)(2, 3)}. We know that
A4 = {(), (1, 2, 3), (1, 3, 2), (2, 3, 4), (2, 4, 3), (1, 2, 4), (1, 4, 2), (1, 3, 4), (1, 4, 3), (1, 2)(3, 4), (1, 3)(2, 4), (1, 4)(2, 3)}.
Since H is a finite and nonempty subset of A4 and closed under the binary operation, it is a subgroup of A4 .
To show it is a normal subgroup. We need to show that aha1 H for all h H and a A4 .
Since conjugation preserves shapes (see Example 7.2.3 in Beachy and Blairs text), aHa1 preserve the
cycle structure of H. Since H contains all of elements of A4 that have this cycle structure, aHa1 = H.
Hence H is normal in A4 .
The definition of solvable is a chain of normal subgroups G = N0 N1 N2 Nn = {e} such that
(1) Ni E Ni1 for i = 1, 2, 3, . . . n
(2) Ni1 /Ni is abelian for i = 1, 2, 3, . . . n
(3) Nn = {e}.
The chain to verify is A4 H {e} where G = N0 , A4 = N1 , H = N2 and N3 = {e}.
(1) H E A4 (just proven) and {e} E H since {e} is the trivial normal subgroups to all subgroups.
(2) By Lagrange, the factor group A4 /H will have three cosets. Since this is a group of prime order,
A4 /H
= Z/3Z. So it is cyclic and therefore abelian.
H/{e} = {(1), (12)(34), (13)(24), (14)(23)} = H. So H is the identity in the factor group and the identity
commutes with itself. So H/{e} is abelian.
(3) N3 = {e}.
Therefore A4 is solvable.//
Dummit & Foote Text Exercise 3.5-11: Prove that 4 has no subgroup isomorphic to Q8 .
Solution: Assume a subgroup H 4 exists such that H
= Q8 .
Q8 contains 6 elements of order 4 namely (i, i, j, j, k, and k). 4 also contains exactly 6 elements of
order 4: the 4-cycles. So H contains all 4-cycles in Sigma4 . Since H is a subgroup, it is closed under the
binary operation. That implies H contains elements of order three (e.g., (1234)(1342) = (143) and
(1234)(1423) = (243)). Q8 has no elements of order 8 (by Lagrange). A contradiction. And #H > 8 since
it contains six elements of order four, one identity, and more than one element of order three. Another
contradiction. Therefore, 4 has no subgroup isomorphic to Q8 .//

MAT7400 Assignment #3

Dummit & Foote Text Exercise 7.1-1 Let R be a ring with 1. Show that (1)2 = 1 in R.
Solution:

and

(1)2 + (1)

= (1)(1) + (1)
= (1)((1) + 1)
= (1)(0)
= 0 (1)2 = 1.

(1) + (1)2

= (1) + (1)(1)
= (1 + (1))(1)
= (0)(1)
= 0 (1)2 = 1.

Dummit & Foote Text Exercise 7.1-2: Prove that if u is a unit in R then so is u.
Solution: Since u is a unit, there exists some v R such that uv = vu = 1. From Exercise 7.1.1, we know
that (1)2 = 1. So 1 = uv = u 1 v = u(1)2 v = u(1)(1)v = (u)(v) and
1 = vu = v 1 u = v(1)2 u = v(1)(1)u = (v)(u). Hence u is a unit.//
Dummit & Foote Text Exercise 7.1-4: Prove that the intersection of any nonempty collection of
subrings of a ring is also a subring.
Solution: Let T
R be a ring and let X be a nonempty T
set of subrings of R.
T
We know that X R is a subgroup. To show that X is closed T
under multiplication, letTa, b X.
Then a, b S for all S X, and xy S for all S X. Hence xy X, and by definition X R is a
subring. Note that the distributive properties did not have to be checked as those are inherited if
multiplicative closure holds.//
Dummit & Foote Text Exercise 7-1-5: Decide which of the following (a)- (f) are subrings of Q:
(a) the set of all rational numbers with odd denominators (when written in lowest terms)
(b) the set of all rational numbers with even denominators (when written in lowest terms)
(c) the set of nonnegative rational numbers
(d) the set of squares of rational numbers
(e) the set of all rational numbers with odd numerators (when written in lowest terms)
(f ) the set of all rational numbers with even numerators (when written in lowest terms).
Solution: (a) Let X be the given set.
Associativtity in X for addition is inherited from Q.
X is closed under addition. If ab , dc X, then b and d are odd. Therefore bd is odd. Next ab + dc = ad+bc
bd
and is equal to some other fraction pq in lowest terms, such that q|bd. Because bd is odd, q must also be
odd. So X is closed under addition.
0
0
a
a
a
0
a
1 is the additive identity element for X since 1 + b = b = b + 1 for all b X.
a
a
a
a
0
0
a
a
0
0
For every b X, b X. Because b + b = b = 1 and b + b = b = 1 , every element of X has an
additive inverse.
So X is a group under addition. if f racab and dc are written in lowest terms and b and d are odd, then
when ac/bd is written in lowest terms, its denominator must divide bd and therefore is odd. Hence X is
closed under multiplication and X is a subring.//
(b) Let X be the given set. If a is odd, then 2a is even. So
closed under addition, X cannot be a subring of Q.

1
2a

X but

1
2a

1
2a

1
a

6 X. Since X is not

(c) This set does not contain additive inverses for all of its elements and therefore it is not a group under
addition. Hence, it cannot be a subring of Q.

(d) This set X, is not closed under addition. ( 31 )2 X but ( 31 )2 + ( 13 )2 = 29 6 X since 2 6 Q and
therefore cannot be in X.

MAT7400 Assignment #3

(e) This set X is not closed under addition. 15 + 51 = 25 6 X but 51 X.


adbc
(f ) Let Y be the given set. If ab and dc are in lowest terms and a and c are even, then ab + c
d =
bd . We
adbc
must have b and d be odd because when reduced to lowest terms, the numerator of bd is even since 2
divides a and b.
Since Y contains 2 = 21 , Y Q is a subgroup. Now if ab and dc Y and in lowest terms, then b and d are
odd. Thus when expressed in lowest terms, 2 must divide ac. Thus Y is closed under multiplication. Now
to check for the multiplicative identity. Assume Y has an identity element dc . Then for all ab we have
ca
a
db = b , so that cab = dab. Thus c = d. Since c must be even and d odd, we have a contradiction. So Y is a
not ring with a multiplicative identity. Y is a ring but not a unital ring. The definition of subring requires
the identity element of the group operation (addition) and Y meets that definition. Y can be considered a
subring of Q.
Dummit & Foote Text Exercise 7.1-7: The center of a ring R is {z R | zr = rz for all r R} (i.e., is
the set of all elements which commute with every element of R). Prove that the center of a ring is a
subring that contains the identity. Prove that the center of a division ring is a field.
Solution: Z(R) contains the zero element (additive identity). 0 Z(R) because 0 r = 0 = r 0 for all
r R Hence, Z(R) is nonempty. If a, b Z(R) and r R, then (a b)r = ar br = ra rb = r(a b). By
the subgroup criterion, Z(R) R. Next abr = arb = rab, so that ab Z(R). Hence by definition of a
subring, Z(R) is a subring.
If R has a 1 (multiplicative identity), then, 1 a = a1 = a for all a R. Hence 1 Z(R).
Next let R be a division ring, and consider its center Z(R). If a Z(R), then by the cancellation law, the
inverse of a is unique. Let the inverse of a be a1 . We know that (r1 )1 = r. Since (ab)(b1 a1 ) = 1, we
have (ab)1 = b1 a1 . Now let r R. a1 r1 = (ra)1 = (ar)1 = r1 a1 . Since r1 R, a1 Z(R).
Since Z(R) is a commutative division ring, it is a field.
Dummit & Foote Text Exercise 7.1-13: An element X in R is called nilpotent if xm = 0 for some
m Z+ . (a) Show that if n = ak b for some integers a and b then ab is a nilpotent element of Z/nZ.
(b) If a Z is an integer, show that the element a Z/nZ is nilpotent if and only if every prime divisor of
n is also a divisor of a. In particular, determine the nilpotent elements of Z/72Z explicitly.
(c) Let R be the ring of functions from a nonempty set X to a field F . Prove that R contains no nonzero
nilpotent elements.
Solution: (a) Let n = ak b, where k 1. Then (ab)k = ak bk = (ak b)bk1 = nbk1 0 mod n.
(b) () Assume a Z/(n) is nilpotent. Then am = nk for some m and k. If p is a prime that divides n,
then p divides am , so that it divides a. Hence every prime that divides n divides a.
() Let n = pb11 pbkk and a = pc11 pckk m, where 1 bi , ci for all i and m is an integer. Let t = max{bi }.
Then at = (pc11 pckk m)t = pc1i t pcki t mt , where ci t bi for each i. Hence at = nd for some integer d, and
we have at 0 mod n.
Since 72 has prime factorization 23 32 and since every prime that divides n must divide a the only nilpotent
elements in Z/nZ are those a such that a is a multiple of 6 (for any representative choice of a):
{0, 6, 12, 18, 24, 30, 36, 42, 48, 54, 50, 66}.
(c) Proof by contradiction. Assume f R is nilpotent. If f 6= 0, then there exists x X such that
f (x) 6= 0. Let m be the least positive integer such that f (x)m = 0. Then f (x)f (x)m1 = 0 and f (x) 6= 0
and f (x)m1 6= 0. We have our desired contradiction as F contains zero divisors. Therefore no nonzero
element of R is nilpotent.//

MAT7400 Assignment #3

Dummit & Foote Text Exercise 7.1-14: Let x be a nilpotent element of the commutative ring R (cf.
the preceding exercise).
(a) Prove that x is either zero or a zero divisor.
(b) Prove that rx is nilpotent for all r R.
(c) Prove that 1 + x is a unit in R.
(d) Deduce that the sum of a nilpotent element and a unit is a unit.
Solution: (a) Let n be the least positive integer such that xn = 0. If n = 1, then x = 0. If n > 1, then
x 6= 0, xn1 6= 0 and x xn1 = 0, then x is a zero divisor by definition of a zero divisor.//
(b) Let x be defined as in part (a). Because R is commutative, (rx)n = rn xn = rn 0 = 0.//
(c) Let x be defined as in part (a). If x = 0, then clearly 1 = 1 + x is a unit in R. So assume x R and
x 6= 0. We know that 1 = (1 xn ) = (1 + x)(1 x + x2 + (1)n1 xn1 ). We also know from the closure
properties of a ring, (1 x + x2 + xn1 ) R. Thus 1 + x is a unit in R.//
(d) Let x be defined as in part (a). If u is any unit, then u + x = u(1 + u1 x) is the product of two units
(using the results of part (b). Hence (u + x) is a unit.//
Dummit & Foote Text Exercise Q
7.1-19: Let I be any nonempty index set and let Ri be a ring for each
i I. Prove that the direct product iI Ri is a ring under componentwise addition and multiplication.
Solution: In Dummit & Foote Text Exercise 5.1-15 we proved that the direct product of groups is a group
and it certainly holds for abelian groups. We need to show associativity of multiplication and the left and
right distributive properties.
Q
Q
Q
Q
IfQ( ai ),
), ( ci ) Q I RQ
i , then
Q( biQ
Q
Q
Q
Q
Q
Q
Q
( ai )(( bi )( ci )) = ( ai )( (bi ci )) = ai (bi ci ) = (ai bi )ci = ( ai bi )( ci ) = (( ai )( bi ))( ci ),
so multiplication is associative.
Q
Q
Q
Q
Q
Q
Q
Q
Q
Next,
bi ) + (Q ci )) = ( ai )( (bi + ci )) = ai (bi + ci ) = (ai bi + ai ci ) = ( ai bi ) + ( ai ci ) =
Q ( Q ai )(( Q
( ai )( bi ) + ( ai )( ci ). So, multiplication distributes over addition on the left.
Q
Q
Q
Q
Q
Q
Q
Q
Q
((Q ai )Q
+ bi )(Q ci ) =
Q( (ai + bi ))( ci )) = (ai + bi )(ci ) = (ai ci + bi ci ) = ( ai ci ) + ( bi ci ) =
( ai )( ci ) + ( bi )( ci ). So, multiplication distributes over addition on the right as well.
Q
Thus I Ri is a ring under componentwise addition and multiplication.//
Dummit & Foote Text Exercise 7.1-26: Let K be a field. A discrete valuation on K is a function
v : K Z satisfying
(i) v(ab) = v(a) + v(b) (i.e., vis a homomorphism from the multiplicative group of nonzero elements of K
to Z,
(ii) v is surjective, and
(ii) v(x + y) min{v(x), v(y)} for all x, y K with x + y 6= 0. The set R = {x K | v(x) 0} {0} is
called the valuation ring of v.
(a) Prove that R is a subring of K which contains the identity. (In general, a ring R is called a discrete
valuation ring if there is some field K and some discrete valuation v on K such that R is the valuation ring
of v.)
(b) Prove that for each nonzero element x K either x or x1 is in R.
(c) Prove that an element x is a unit of R if and only if v(x) = 0.
Solution: v is an epimorphism (surjective homomorphism) since we are given thatv is is surjective and it
has the homomorphic property for addition. So if x R, then v(x) = v(1 x) = v(1) + v(x). This implies
v(1) = 0 by cancellation. We can use this to show v(x) = v(x) as follows:
0 = v(1) = v((1)(1)) = v(1) + v(1) implies v(1) = v(1) and that in turn implies v(1) = 0. So
if x R, then v(x) = v((1)x) = v(1) + v(x) = v(x). Hence, v(x) = v(x). This fact is necessary for
part (a).
(a) By definition, 0 R and that implies R is nonempty. Apply the subgroup criterion. Let x, y R and
consider x y. If x 6= 0 and y = 0, then v(x y) = v(x) 0. If x = 0 and y 6= 0, then

MAT7400 Assignment #3

v(x y) = v(y) = v(y) 0. If x = y = 0, then x y = 0 R. If x, y 6= 0, then either x y = 0 R or


x y 6= 0, and then v(x y) min(v(x), v(y)) = min(v(x), v(y)) 0.
Now to check for multiplicative closure. If x, y R and xy = 0, then xy R. If xy 6= 0, then
v(xy) = v(x) + v(y) 0, so that xy R. Thus R is a subring of K. We showed earlier that v(1) = 0, so
that 1 R.
(b) Assume x K is nonzero. So, 0 = v(1) = v(xx1 ) = v(x) + v(x1 ). Hence, v(x) = v(x1 ), and either
v(x) or v(x1 ) is nonnegative and therefore either x orx1 R. (c) () If u R is a unit then u1 R .
From part (b), v(u) = v(u1 ), and both v(u) and v(u1 ) are nonnegative. Hence v(u) = v(u1 ) = 0.
() If v(u) = 0, then 0 = 0 = v(u) = v(u1 ), so that u1 R and u R is a unit.//
Dummit & Foote Text Exercise 7.1.27: A specific example of a discrete valuation ring (cf. the
preceding exercise) is obtained when p is a prime, K = Q and
a
a
c
= where = p , p 6 |c and p 6 |d.
vp : Q Z by vp
b
b
d
Prove that the corresponding valuation ring R is the ring of all rational numbers whose denominators are
relatively prime to p. Describe the units of this valuation ring.
Solution: We are asked to prove that R = {a/b Q | a = 0 or gcd(b, p) = 1}.
(R {a/b Q | a = 0 or gcd(b, p) = 1}). Assume that vp ( ab ) 0, and let a = pm c and b = pn d, where p
does not divide c or d. By definition, vp ( ab ) = m n 0. Take the case that ab is in lowest terms then
either m or n is 0. If n 6= 0, then m = 0, and vp ( ab ) < 0, a contradiction. Hence n = 0 and gcd(b, p) = 1.
({a/b Q | a = 0 or gcd(b, p) = 1} R). If ab Q and p does not divide b, then
Hence vp (f racab) 0, and we have ab R.//

a
b

= p ) dc for some 0.

Dummit & Foote Text Exercise 7.2-2 Let p(x) = an xn + anl xnl + + a1 x + a0 be an element of
the polynomial ring R[x]. Prove that p(x) is a zero divisor in R[x] if and only if there is a nonzero b R
such that bp(x) = 0. [Let g(x) = bm xm + bm1 xml + + b0 be a nonzero polynomial of minimal degree
such that g(x)p(x) = 0. Show that bm an = 0 and so an g(x) is a polynomial of degree less than m that also
gives 0 when multiplied by p(x). Conclude that an g(x) = 0. Apply a similar argument to show by
induction on i that ani g(x) = 0 for i = 0, 1, . . . , n and show that this implies bm p(x) = 0.]
Solution: ) If bp(x) = 0 for some nonzero b R, then letting b(x) = b states that p(x) is a zero divisor.
Pm
() Assume p(x) is a zero divisor. For some q(x) = i=0 bi xi , p(x)q(x) = 0. We may choose q(x) to have
minimal degree among the nonzero polynomials with this property.
Now for the induction argument to show that ai q(x) = 0 for all 0 i n. I obtained help for this part
of the problem.

Pn+m P
The base case: p(x)q(x) = k=0
a
b
xk = 0. The coefficient of xn+m in this product is an bm
i
j
i+j=k
on the left side and 0 on the right side. Thus an bm = 0. Now an q(x)p(x) = 0, and the coefficient of xm in q
is an bm = 0. Thus the degree of an q(x) is strictly less than that of q(x). Since q(x) has minimal degree
among the nonzero polynomials which multiply p(x) to 0, an q(x) = 0. So, an bi = 0 for all 0 i m.
For the inductive step,
suppose that
for some 0 s < 4, we have ar q(x) = 0 for all s < r n. Now

Pn+m P
P
k
m+s
p(x)q(x) = k=0
is i+j=m+s ai bj on side side of the
i+j=k ai bj x = 0. The coefficient of x
P
equation and it is 0 on the other. Thus i+j=m+s ai bj = 0. By the induction hypothesis, if i s, then
ai bj = 0. Hence all terms such that i s are zero. If i < s, then we must have j > m, a contradiction.
Thus we have as bm = 0. As in the base case, as q(x)p(x) = 0 and as q(x) has degree strictly less than that
of q(x), so that by minimality, as q(x) = 0.
By induction, ai q(x) = 0 for all 0 i n. In particular, ai bm = 0. Hence bm p(x) = 0.

MAT7400 Assignment #3

Dummit & Foote Text Exercise 7.2-3: Define the set R[[x]] of formal power series in the
indeterminate x with coefficients from R to be all formal infinite sums

an xn = a0 + a1 x + a2 x2 + a3 x3 + .

n=0

Define addition and multiplication of power series in the same way as for power series with real or complex
coefficients i.e., extend polynomial addition and multiplication to power series as though they were
polynomials of infinite degree:

an x +

n=0

bn x =

n=0

an xn

n=0

(an + bn )xn

n=0

bn xn =

n=0

n
X
X
n=0

!
xn .

ak bnk

k=0

(The term formal is used here to indicate that convergence is not considered, so that formal power series
need not represent functions on R.)
(a) Prove that R[[x]] is a commutative ring with 1.
(b) Show that 1P x is a unit in R[[x]] with inverse 1 + x + x2 + .

(c) Prove that n=0 an xn is a unit in R[[x]] if and only if a0 is a unit in R.


P
P
P
Solution: (a) Let r = n=0 an xn , s = n=0 bn xn , and v = n=0 cn xn .
Addition is Associative:
P
P
P
(r + s) + v = (( n=0 an xn ) + ( n=0 bn xn )) + ( n=0 cn xn )
P

+ bn )xn ) + (

+ bn ) + cn )xn

+ (bn + cn ))xn

an xn ) + (

an xn ) + ((

n=0 (an

n=0 ((an
n=0 (an

n=0

n=0

n=0 cn x

n=0 (bn

+ cn )xn )

n=0 bn x

)+(

n=0 cn x

= r + (s + v).
Addition is Commutative:
r+s =

!
an x

n=0

!
bn x

an x

n=0

(an + bn )xn

n=0

(bn + an )xn

n=0

X
n=0

=s+r
P

n=0

0 xn = 0 is the additive identity.

!
bn x

X
n=0

))

MAT7400 Assignment #3

r+0

0+r

n=0

an xn ) + (

n=0 (an

n=0

n=0

n=0

0 xn )

+ 0)xn

an xn ) =

n=0 (0

+ an )xn

0 xn ) + (

n=0

an xn )

P
R[[x]] contains left and right inverses. Define r R[[x]] as r = n=0 (an )xn . Then we have
P
P
r + r = ( n=0 an xn ) + ( n=0 (an )xn )
=

n=0 (an

n=0

n=0

an )xn

0 xn

0 xn

n=0 (an

+ an )xn

n=0 (an )x

)+(

n=0

an xn )

= r + r.
Hence the left and right inverses coincide. r is the additive inverse for r.
I had a lot of help sorting this one out. The definition of the product is identical to that of what electrical
engineers know as convolution. Multiplication is Associative:
!
!!
!

X
X
X
n
n
n
(rs)v =
an x
bn x
cn x
n=0

ai bj x

t+k=n

cn x

ai bj ck xn

i+j=t

n=0

n=0

n=0

i+j=n

n=0

n=0

n=0

t+k=n i+j=t

ai bj ck xn

MAT7400 Assignment #3

10

i+j+k=n

ai bj ck xn

n=0

ai bj ck xn

n=0

i+s=n j+k=s

ai

n=0

i+s=n

!
an xn

j+k=n

!
n

n=0

bj c k x n

n=0

an x

bj ck xn

j+k=s

n=0

!
n

bn x

n=0

!!
cn x

n=0

= r(sv).
Distributivity:
r(s + v)

!
an x

n=0

!
an x

ai bj + ai cj xn

i+j=n

ai bj xn +

i+j=n

!
an x

n=0

= rs + rv.

ai cj xn

i+j=n

ai bj +

i+j=n

n=0

n=0

(bn + cn )x

ai (bj + cj ) xn

!
n

i+j=n

n=0

cn x

n=0

!!
n

n=0

bn x

n=0

!
n

n=0

n=0

X
n=0

!
n

bn x

n=0

i+j=n

n=0

!
an x

ai cj xn

X
n=0

!
cn x

MAT7400 Assignment #3

(r + s)v

11

!
an x

n=0

!
(an + bn )x

ai cj + bi cj xn

i+j=n

n=0

ai cj +

i+j=n

ai cj xn +

i+j=n

!
an x

bi cj xn

i+j=n

n=0

(aj + bj )ci xn

cn x

i+j=n

n=0

!
n

cn x

n=0

!
n

n=0

n=0

bn x

n=0

n=0

!!
n

!
cn x

n=0

i+j=n

!
bn x

Hence multiplication distributes over addition on the left and right.


If R is commutative for multiplication, then so is R[[x]]:
!
!

X
X
n
n
rs =
an x
bn x

n=0

i+j=n

bj ai xn

X
n=0

n=0

ai bj xn

n=0

sr.

j+i=n

!
bn x

X
n=0

!
an x

bi cj xn

X
n=0

= rv + sv.

n=0

n=0

n=0

n=0

!
cn x

MAT7400 Assignment #3

12

P
If R has a multiplicative identity, then so does R[[x]]. Define 1 = n=0 en xn by e0 = 1 and ei+1 = 0.
That gives us
!
!

X
X
n
n
r1 =
an x
en x
n=0

ai ej xn

n=0

n=0

i+j=n

an e0 xn

n=0

an xn

n=0

= r
1r

!
en x

n=0

an x

n=0

!
n

n=0

ej ai xn

i+j=n

e0 an xn

n=0

an xn

n=0

= r
Hence R[[x]] has the multiplicative identity 1.
P
P
(b). Let 1 x = n=0 bn xn where b0 = 1, b1 = 1, and bi = 0 for i 2. Let r = n=0 xn . Taking the
product gives

!
!

X
X
X
X

(1 x)r =
bn x n
xn =
bi xn .
n=0

So if n 1, then

n=0

n=0

i+j=n

di = 0, and d0 = 1. Hence (1 x)r = 1.


P
(c) () Suppose u R[[x]] is a unit, with inverse u1 = n=0 an xn . So,

!
!

X
X
X
X

1 = uu1 =
an xn
an xn =
ai aj xn .
i+j=n

n=0

n=0

n=0

i+j=n

The coefficient of x in this power series is 1 on the left hand side and a0 a0 on the right hand side. Thus
a0 a0 = 1 R. We also have

!
!

X
X
X
X

1 = u1 u =
an xn
an xn =
ai aj xn .
n=0

n=0

n=0

i+j=n

MAT7400 Assignment #3

13

Again, the coefficient of x0 in this power series is 1 on the left hand side and a0 a0 on the right hand side.
Hence a0 a0 = 1 R and a0 is a unit in R.
I had help on the following part (could not keep the indices on the
consistent).
Psums

() Suppose
a

R
is
a
unit,
with
a
a
=
a
a
=
1.
Define
u
=
b
xn with b0 = a0 and
0
0
0
0
0
n
n=0
P
bk+1 = a0 i+j=k+1,jk ai bj . Then

!
!

X
X
X
X

uu =
an xn
bn x n =
a i bj x n
n=0

If n = 0,

i+j=n

n=0

n=0

ai bj = a0 a0 = 1. If n 1, then

X
i+j=n

i+j=n

ai bj = a0 bn +

ai bj = a0 a0

i+j=n,j<n

ai bj +

i+j=n,j<n

ai bj = 0.

i+j=n,j<n

P
Thus uu = 1. Similarly, we can see that if v = n=0 cn xn where c0 = a0 and

X
ck+1 =
ai cj (a0 )
i+j=n,jk

then vu = 1. Since u has both a left and a right inverse, they are equal (see part (a)) and u is a unit in
R[[x]].//
Dummit & Foote Text Exercise 7.2-10: Consider the following elements of the integral group ring
ZS3 : = 3(12) 5(23) + 14(123) and = 6(1) + 2(23) 7(132).
(where (1) is the identity of S3 ). Compute the following elements:
(a) + , (b) 2 3, (c) , (d) , (e) 2 .
Solution: (a) + = 6(1) + 3(1 2) 3(2 3) + 14(1 2 3) 7(1 3 2)
(b) 2 3 = 18(1) + 6(1 2) 16(2 3) + 28(1 2 3) + 21(1 3 2)
(c) = 108(1) + 81(1 2) 21(1 3) 30(2 3) + 90(1 2 3)
(d) = 108(1) + 18(1 2) + 63(1 3) 51(2 3) + 84(1 2 3) + 6(1 3 2)
(e) 2 = 34(1) 70(1 2) 28(1 3) + 42(2 3) 15(1 2 3) + 181(1 3 2)
Dummit & Foote Text Exercise 7.2-11: Repeat the preceding exercise under the assumption that the
coefficients of and are in Z/3Z (i.e., , Z/3ZS3 ).
Solution: Evidently, (a) + = 2(1 2 3) + 2(1 3 2)
(b) 2 3 = 2 = 2(2 3) + (1 2 3)
(c) = 2(1 2) + 1(1 2 3)
(d) = 1(1 3) + 2(1 3 2)
(e) 2 = 1(1) + 2(1 2) + 2(1 3) + 1(1 3 2)
Dummit & Foote Text Exercise 7.2-12: Let G = {g1 , . . . , gn } be a finite group. Prove that the
element N = g1 + g2 + + gn is in the center of the group ring RG (cf. Exercise 7, Section 1).
Solution: The center of a ring R is {z R | r R, zr = rz}. That is, the elements of R that commute
with every element of r. In the group ring RG, for every g G we have
gN = g(g1 + g2 + + gn ) = gg1 + gg2 + + ggn , but left multiplication by g is a permutation of the
element of G, so G = {gg1 , gg2 , . . . , ggn }, hence gN = N . Let r1 g1 + + rn gn RG be arbitrary. Then
applying the definition of multiplication in a group product we obtain
N (r1 g1 + + rn gn ) = r1 N g1 + + rn N gn .

MAT7400 Assignment #3

14

r1 N g1 + + rn N gn = r1 N + + rn N since N g = N .
Then r1 N + + rn N = r1 g1 N + + rn gn N since gN = N .
Then by the distributive property, we have r1 g1 N + + rn gn N = (r1 g1 + + rn gn )N .
So N (r1 g1 + + rn gn ) = (r1 g1 + + rn gn )N implies N is in the center of RG. //
Dummit & Foote Text Exercise 7.3-2: Prove that the rings Z[x] and Q[x] are not isomorphic.
Solution: Suppose : Q[x] Z[x] is a ring homomorphism. Then since (1) = 1 by definition of a ring
homomorphism,
(2) = (1 + 1) = (1) + (1) = 1 + 1 = 2.
We also have
1 = (1) = (2 1/2) = 2 (1/2).

Thus (1/2) Z[x] . By by Proposition 7.2.4(2) Z[x] = Z = {+1, 1} . Since 2 1 6= 1 and 2 1 6= 1,


we have a contradiction. Hence no such homomorphism exists.//
Dummit & Foote Text Exercise 7.3-4: Find all ring homomorphisms from Z to Z/30Z. In each case
describe the kernel and the image.
Solution: A homomorphism : Z Z/30Z requires that (x)(x 1) = x (1). We need to determine
how many images of 1 are possible since each of them defines the map. We know that
(x + y) = (x + y)(1) = x(1) + y(1) = (x) + (y). Also, (xy) = xy(1) while
(x)(y) = x(1)y(1) = xy(1)2 This implies that we have a homomorphism if and only if (1) = (1)2
in Z/30Z. The eight elements of Z/30Z which meet this requirement are {0, 1, 6, 10, 15, 16, 21, 25}.
Map

Kernel

Image

(1) 7 0

{0}

(1) 7 1

h30i

h1i

(1) 7 6

h5i

h6i

(1) 7 10

h3i

h10i

(1) 7 15

h2i

h15i

(1) 7 16

h15i

h16i = h2i

(1) 7 21

h10i

h21i = h3i

(1) 7 25

h6i

h25i = h5i

MAT7400 Assignment #3

15

Dummit & Foote Text Exercise 7.3-5: Describe all ring homomorphisms from the ring Z Z to Z.
Solution: Z Z has two generators (0, 1) and (1, 0). So any homomorphism has to send generators to
generators. If : mathbbZ Z Z is a ring homomorphism then is determined by where it sends the
two generators, say (1, 0) = a and (0, 1) = b.
So (1, 1) = (1, 0) + (0, 1) = a + b and (1, 1) = ((1, 1)(1, 1)) = (1, 1)(1, 1) = (a + b)2 . This implies
that a + b = 0 or a + b = 1.
Next we have for all (s, t) Z Z, (s, t) = (s 1, t 1) = sa + yb and
(s, t) = ((1, 1)(s, t)) = (a + b)(sa + sb). So (s, t) = sa + tb = (a + b)(sa + tb).
If a + b = 0, (s, t) = 0 (s, t) Z Z. this implies = 0.
The remaining case is a + b = 1. Since is a ring homomorphism,
((s, t)(u, v)) = (s, t)(u, v) = (as + bt)(au + bv) and ((s, t)(u, v)) = (su, tv) = asu + btv. This implies
asu + btv = a2 su + (sv + tu)ab + b2 tu for all integers s, t, v and v.
If t = v = 0 and s, u 6= 0, then asu = a2 su. Hence a2 = a. Since a is an integer, we have a {0, 1}. That
implies (a, b) {(1, 0), (0, 1)}.
So we have three ring homomorphisms (maps):
1. (1, 0) 7 0 and (0, 1) 7 0.
2. (1, 0) 7 1 and (0, 1)0.
3. (1, 0) 7 0 and (0, 1)1.
Dummit & Foote Text Exercise 7.3-6: Decide which of the following are ring homomorphisms from
M2 (Z) to Z:


a b
(a)
7 a (projection onto the 1,1 entry.)
c d 
a b
(b)
7 a + d (the trace of the matrix)
 c d
a b
(c)
7 ad bc (the determinant of the matrix).
c d




1 1
2 1
Solution: (a) If A =
, then (AA) =
= 2 and (A)(A) = 1, Since they are not equal,
1 0
1 0
this mapping is not a ring homomorphism.




1 1
2 1
(b) If A =
, then (AA) =
= 3 and (A)(A) = 1.Since they are not equal, this
1 0
1 0
mapping is not a ring homomorphism.




1 0
0 0
(c) If A =
and B =
, then (A + B) = 1 and (A) + (B) = 0. Since they are not equal,
0 0
0 1
this mapping is not a ring homomorphism.

a
Dummit & Foote Text Exercise 7.3-7: Let R = {
0
triangular matrices. Prove that the map
: R Z Z defined by


b
| a, b, d Z} be the subring of all upper
d

a
0


b
7 (a, d)
d

is a surjective ring homomorphism and describe its kernel.






a1 b1
a2 b2
Solution: Let A, B R with A =
and B =
.
0 d1
0 d2
is a ring homomorphism since

MAT7400 Assignment #3

16


(A + B)

 
b1
a
+ 2
d1
0

(a1 + a2 , d1 + d2 )

(a1 + a2 , d1 + d2 )

(A) + (B)



and
(AB)

a1
0

a1
0


a1
0

b1
d1

b1
d1




a2
0

(a1 a2 , d1 d2 )

(a1 , d1 )(a2 , d2 )


a1
0

b1
d1



b2
d2




a2
0

b2
d2

b2
d2






a2
0

b2
d2



= (A) (B).


a 0
2
is surjective since if (a, d) Z , then
= (a, d).
0 d





a b
a b
0
If
ker , then
= (a, d) = (0, 0) which implies a = d = 0. Since
0 d
0
d
0



0 b
bZ .
ker =
0 0

b
0


= (0, 0),

Dummit & Foote Text Exercise 7.3-8: Decide which of the following are ideals of the ring Z Z.
(a) {(a, a) | a Z}
(b) {(2a, 2b) | a, b Z}
(c) {(2a, 0) | a Z}
(d) {(a, a) | a Z}.
Solution:Let R = Z Z for all four of these subproblems. (a) Let R = Z Z and S = {(a, a) | a Z}. S
is not an ideal of R because S does not absorb R. (1, 1) S but (1, 0)(1, 1) = (1, 0)
/ S.
(b). We need to verify that S = {(2a, 2b) | a, b Z} is a subring of R and absorbs R. Let
(2a1 , 2b1 ), (2a2 , 2b2 ) I and (x, y) R. (0, 0) I, so that I is not empty.
(2a1 , 2b1 ) (2a2 , 2b2 ) = (2(a1 a2 ), 2(b1 b2 )) S, so that S is an additive subgroup.
(2a1 , 2b1 )(2a2 , 2b2 ) = (4a1 a2 , 4b1 b2 ) S shows S is closed under multiplication, so that S is a subring.
(2a1 , 2b1 )(x, y) = (2a1 x, 2b1 y) S, so that S absorbs R. Since R is a commutative ring, we dont need to
calculate absorption on the right. Hence S is an ideal of R.
(c). We need to show that S = {(2a, 0) | a Z} is a subring and absorbs R on one side (since R is
commutative. . (Absorption on one side is sufficient since R is commutative.) Let (2a1 , 0), (2a2 , 0) I and
(x, y) R. (0, 0) S, so that S is not empty. (2a1 , 0) (2a2 , 0) = (2(a1 a2 ), 0) S, so that S is an
additive subgroup. (2a1 , 0)(2a2 , 0) = (4a1 a2 , 0) S shows I is closed under multiplication, so that S is a
subring. (2a1 , 0)(x, y) = (2a1 x, 0) S, so that I absorbs R. Hence S is an ideal of R.
(d) The set S = {(a, a) | a Z} is not closed under the multiplication operation: (1, 1) S but
(1, 1)(1, 1) = (1, 1)
/ S. So S is not a subring and hence cannot be an ideal.

MAT7400 Assignment #3

17

Dummit & Foote Text Exercise 7.3-10: Decide which of the following are ideals of the ring Z[x]:
(a) The set of all polynomials whose constant term is a multiple of 3
(b) The set of all polynomials whose coefficient of x2 is a multiple of 3
(c)The set of all polynomials whose constant term, coefficient of x, and coefficient of x2 are all zero
(d) Z[x2 ] (the set of all polynomials in which only even powers of x appear)
(e) The set of all polynomials whose coefficients sum to zero
(f ) The set of polynomials p(x) such that p0 (0) = 0, where p0 (x) is the usual first derivative of p with
respect to x.
Solution: (a) Let S be the given set and = 3a + xp(x), = 3b + xq(x) S and = r + xt(x) Z[x].
We know that 0 S and = 3(a b) + x(p(x) q(x)) S. This implies S is a subgroup. S is a
subring since = 9ab + x(3bp(x) + 3aq(x) + xp(x)q(x)) S. Lastly, S is closed under multiplication since
= 3ar + x(3at(x) + rp(x) + xp(x)t(x)) S. Hence S is an ideal.
(b) This set is not closed under multiplication since x S but xx = x2
/ S. Hence S is not a subring and
therefore cannnot be an ideal.
(c) Let S be the given set and = x3 p(x), = x3 q(x) S and c(x) Z[x]. We know that 0 S, and
= x3 (p(x) q(x)) S. So S is a subgroup. = x6 p(x)q(x) S shows that S is closed under
multiplication. Hence t S is a subring. S absorbs Z[x]. since c(x) = x3 p(x)c(x) S. Hence, S is an ideal.
(d) Sinc3 x2 is in this set, but x2 x = x3 is not. Since this subset does not absorb Z[x] on the left, it is not
an ideal.
(e) A polynomial p(x) is in this set S when p(1) = 0. Suppose p, q S and r Z[x]. We know that 0 S.
Since (p q)(1) = p(1) q(1) = 0, implies p q S, S is subgroup. Also, since (pq)(1) = (1), S is closed
under multiplication and hence S is a subring. Since (pr)(1) = p(1)r(1) = 0 r(1) = 0, S absorbs Z[x].
Hence S is an ideal.
(f ) Let S be the given subset and let p(x) = x2 a and q(x) = x. Since p0 (x) = 2x, p S. But
(qp)0 = q 0 p + p0 q, so that (qp)0 (0) = q 0 (0)p(0) + p0 (0)q(0) = p(0) = a. Since S does not absorb Z[x] on the
left, it is not an ideal.
Dummit & Foote Text Exercise 7.3-11: Let R be the ring of all continuous
real valued functions on
R1
the closed interval [0, 1]. Prove that the map : R R given by (f ) = 0 f (x)dx is a homomorphism of
additive groups but not a ring homomorphism.
Solution: We do remember from calculus that integration is a linear operation. That is,
Z 1
Z 1
Z 1
Z 1
(f + g) =
(f + g)(x)dx =
f (x) + g(x)dx =
f (x)dx +
g(x)dx = (f ) + (g)
0

. So, is an additive group homomorphism.


does not meet the homomorphic property for multiplication. For example, (x3 ) = 1/4 while
(x)3 = 1/8. //
Dummit & Foote Text Exercise 7-3.26: The characteristic char(R) of a ring R is the smallest positive
integer n such that 1 + 1 + + 1 = 0 (n times) in R; if no such n exists, then the characteristic of R is
said to 0. For example Z/(nZ) is a ring of characteristic n for each positive integer n and Z is a ring of
characteristic 0.
(a) Prove that the map Z R defined by

if k > 0
1 + 1 + + 1(k times)
0
if k = 0
k 7

1 1 1 (k times) if k < 0
is a ring homomorphism whose kernel is nZ, where n is the characteristic of R (this explains the use of the
terminology characteristic 0 instead of the archaic phrase characteristic for rings in which no sum of
1s is zero).

MAT7400 Assignment #3

18

(b) Determine the characteristics of the rings Q, Z[x], Z/nZ[x].


(c) Prove that if p is a prime and if R is a commutative ring of characteristic p, then (a + b)p = ap + bp for
all a, b R.
Solution: (a). I had help with this part. My approach to the induction argument wasnt as clean as this
one. We will show that (a + b) = (a) + (b) for nonnegative b by induction. For the base case b = 0, we
have (a + 0) = (a) = (a) + 0 = (a) + (0). Now assume that for some b 0, for all a,
(a + b) = (a) + (b). If a + b > 0, then
(a + (b + 1)) = ((a + b) + 1) = (a + b) + 1 = (a) + (b) + 1 = (a) + (b + 1). Hence induction
hypothesis holds for all positive b. Note the case where a + b < 0 is handled by the next case. Assume
b < 0. If a 0, then (a + b) = (b + a) = (b) + (a) = (a) + (b). If a < 0, thena + b < 0, and we have
(a + b) = (a b) = ((a) + (b)) = (a) (b) = (a) + (b). Therefore for all integers a
and b, (a + b) = (a) + (b).
Again we use induction to show that (ab) = (a)(b) for the non-negative case. For the base case,
(a0) = (0) = 0 = (a) 0 = (a)(0). Now assume that the result holds for all a, for some b 0. Then
(a(b + 1)) = (ab + a) = (ab) + (a) = (a)(b) + (a) = (a)((b) + 1) = (a)(b + 1). Hence since the
induction hypothesis, for all a and for all non-negative b. For the negative case, assume b < 0. Then
(ab) = ((a)(b)) = (a)(b) = ((a))((b)) = (a)(b). Thus is a ring homomorphism.
Let n = char R. (ker nZ).
If a ker then (a) = 0. Assume that n does not divide a. Then by the division algorithm, a = nq + r
for some r with 0 < r < n. We are given (n) = 0 by definition and that n is minimal with this property.
Since 0 = (a) = (qn + r) = (r), we have a contradiction in that n is not miminal. Thus n divides a, and
a nZ.
(nZ ker).
If a nZ, then a = nb for some integer b. Since (n) = 0 , (a) = (nb) = (n)(b) = 0 (b) = 0. This
implies a ker.//
(b) Q has no non-identity element whose additive order is finite. So its characteristic is zero. If we select
the inclusion map, it satisfies the requirements defined in part (a). Let the inclusion map. Then,
(0) = 0 = (0), for a 0, (a + 1) = (a) + 1 = (a) + 1 = a + 1 = (a + 1), and
f ora < 0, (a) = (a) (a) = (a) = a = (a). Since the inclusion map is injective, its kernel is
trivial. Hence charQ = 0.
For Z[x], as with Q, let = and the result follows: charZ[x] = 0.
For Z/nZ[x], (n) = n = 0. This implies that nZ ker. Next, let a ker andassume that n 6 |a. By the
division algorithm, a = bn + r where 0 < r < n. Since is a homomorphism,
0 = (bn + r) = (bn) + (r) = (b)(n) + (r) = (r). This is a contradiction of the minimality of n.
Therefore r = 0. This implies n | a and ker nZ. Therefore charZ/nZ = n.
(c) Let R be a commutative ring with characteristic p, and let a, b R. If p is a prime and 0 < k < p, then
k! does not divide p nor does (p k)! divide p!. But by definition of the factorial, p divides p!. So p divides
p!
p
0 < k < p. Applying the binomial theorem and the fact that R has characterstic p
k = k! (n k)! forP
p
gives us (a + b)p = k=0 kp ak bpk = ap + bp .
Dummit & Foote Text Exercise 7.3-28: Prove that an integral domain has characteristic p, where p is
either a prime or 0 (cf. Exercise 26) .
Solution: Proof by contradiction. Assume that R has a characteristic n and n is composite. This implies
n = pq for some integers p, q. Let be the homomorphism from Exercise 7.3-26 which takes an integer
k Z to a sum (k-times) of 1 or -1. This implies (a) and (b) be non-zero. But (a)(b) = (n) = 0 and
that implies (a) and (b) are zero divisors. This contradicts our assumption that R is an integral domain.
Hence, the characteristic of R is not composite. Therefore it is prime or it is zero.//

MAT7400 Assignment #3

19

Dummit & Foote Text Exercise 7.3-29: Let R be a commutative ring. Recall (cf. Exercise 13, Section
1) that an element x R is called nilpotent if xn = 0 for some n Z+ . Prove that the set of nilpotent
elements form an ideal called the nilradical of R denoted by N(R) . [Hint: Use the Binomial Theorem to
show N(R) is closed under addition.]
Solution: If x, y N(R), then for some positive integers n and m, xn = y m = 0. Since we are given the
hint to use the Binomial Theorem, consider (x + y)n+m . The Binomial Theorem tells us
(x + y)n+m =

n+m
X
k=0


n + m k n+mk
x y
.
k

If k n, xk = 0. If k < n, then y n+mk = 0. Hence (x + y)n+m = 0 and x + y N(R). So this set is closed.
The set has inverses for its elements since we have (x)n = (1)n xn = 0, so that x N(R). The set has
an identity since 0 = 01 N(R), N(R) is an additive subgroup of R.
Since R is commutative, if r R then (rx)n = rn xn = 0 and (xr)n = xn rn = 0. So, N(R) absorbs R on the
left and the right. Hence N(R) an ideal.//
Dummit & Foote Text Exercise 7.3-30: Prove that if R is a commutative ring and N(R) is its
nilradical (cf. the preceding exercise), then zero is the only nilpotent element of R/N(R) i.e., prove that
N(R/N(R)) = 0.
Solution: From Exercise 7.3-29, we know that N(R) is an ideal of R. If x + N(R) N(R/N(R)), then for
some positive integer n, (x + N(R))n = xn + N(R) = N(R). Hence xn N(R). Then for some positive
integer m, (xn )m = xnm = 0. Hence x N(R). This implies that x + N(R) = N(R). That is x = 0. In
other words, the additive subgroup N(R/N(R)) contains only one element, the additive identity. //
Dummit & Foote Text Exercise 7.3-34: Let I, J be ideals of R.
(a) Prove that I + J is the ideal of R containing both I and J.
(b) Prove that IJ is an ideal contained in I J.
(c) Give an example where IJ 6= I J.
(d) Prove that if R is commutative and if I + J = R, then IJ = I J.
Solution: I had help with this problem. (a) Let a1 + b1 , a2 + b2 I + J for some ai I and bi J. Then
(a1 + b1 ) (a2 + b2 ) = (a1 a2 ) + (b1 b2 ) I + J since I and J are closed under subtraction. Let r R.
Then, r(a + b) = ra + rb I + J since I and J absorb R on the right. (a + b)r I + J since I and J
absorb R on the left. Thus I + J is an ideal of R. Now, let K be an ideal of R that contains I and J. That
is, I, J K. If a + b I + J, then a + b K since K is closed under addition. So I + J K.
P
P
(b) Let x, y IJ, with x =
ai bi and y P
=
ci di , where ai , ci I and bi , di J. We have x + y IJ.
Let r R. Since I is an ideal of R, rx = (rai )bi IJ. Similarly, xr IJ. Hence IJ is an ideal of R.
P
Again let x = ai bi . Since ai I and I is an ideal, ai bi I, and thus x I. Similarly, x J. Thus
IJ I J.
(c) If R = Z, I = 2Z and J = 4Z, then (2Z)(4Z) = 8Z 6= 2Z 4Z = 4Z.
(d) Assume I + J = R and R is commutative. Part (c) of this exercise tells us that IJ I J. We have
(I J)(I + J) = (I J)R = I J. If a I J, d = c(a + b) for some a I and b J and c I J. Then
d = ca + cb = ac + cb IJ. Hence IJ = I J.//
Dummit & Foote Text Exercise 7.4-4: Assume R is commutative. Prove that R is a field if and only
if 0 is a maximal ideal.
Solution: () Assume R is a field and let I be an ideal of R which properly contains zero. This implies
that there exists an element x 6= 0 in I. Then a is a unit since R is field. Proposition 9 tells us that I = R.
Hence, R is the only ideal of R that properly contains zero. This implies that 0 is a maximal ideal in R.

MAT7400 Assignment #3

20

() Assume that zero is a maximal ideal in R. Let x R and x 6= 0. Since zero is maximal and since R is
given to by commutative, (a) = R. Note that R contains the multiplicative identity, 1. Therefore, exists
elements y and Z inR such that xy = za = 1. This implies that x has a two-sided inverse. Therefore, every
nonzero element in R is invertible and that implies R is a field. //
Dummit & Foote Text Exercise 7.4-7: Let R be a commutative ring with 1. Prove that the principal
ideal generated by x in the polynomial ring R[x] is a prime ideal if and only if R is an integral domain.
Prove that (x) is a maximal ideal if and only if R is a field.
Solution: If R is a commutative ring then : R[x] R defined by (p(x))P
= p(0) is an surjective
P ring
homomorphism and ker = (x). It is clear that is surjective. Let p(x) = ai xi and q(x) = bi xi . We
then have
X

(p + q) =
(ai + bi )xi = a0 + b0 = (p) + (q)
and

X X
(pq) = (
ai bj )xk = a0 b0 = (p)(q)
k

i+j=k

which show that is a ring homomorphism.


To show that ker = (x), assume that p ker. We can write p(x) = a0 + xq(x), and p(0) = a0 = 0. Thus
p(x) = xq(x) (x). So ker (x). Conversely, if p(x) = xq(x), then p(0) = 0 q(x) = 0, and (p) = 0.
(x) ker. Hence ker = (x).
By the First Isomorphism Theorem for rings, we have R[x]/ker = R[x]/(x)
= R. Proposition 13 tells us
that if R[x] is commutative, then the ideal (x) is a prime ideal in R[x] if and only if the quotient ring
R[x]/(x) is an integral domain. Since R[x]/(x)
= R, R is an integral domain if and only if R[x]/(x)
= R is
an integral domain.
Proposition 12 tells us that if R[x] is commutative, then the ideal (x) is maximal if and only if the quotient
ring R[x]/(x) is a field. We have shown by the first isomorphism theorem that R[x]/(x)
= R. So R is a
field if and only if R[x]/(x) is a field.//
Dummit & Foote Text Exercise 7.4-8: Let R be an integral domain. Prove that (a) = (b) for some
elements a, b R, if and only if a = ub for some unit uof R.
Solution: () If (a) = (b), then a (b). This implies that a = ub for some element u R. We also have
b (a) and therefore b = va for some element v R. So a = (uv)a and this implies va = (vu)va. Since
since R is an integral domain we have uv = vu = 1. Hence u is a unit.
() Let a = ub and u R be a unit. Then by Proposition 9 in the text we obtain
(a) = aR = ubR = buR = bR = (b).//
Dummit & Foote Text Exercise 7.4-9: Let R be the ring of all continuous functions on [0, 1] and let I
be the collection of functions f (x) in R with f (l/3) = f (l/2) = 0. Prove that I is an ideal of R but is not a
prime ideal.
Solution: I is an ideal. If f, g I, then (f g)(1/2) = f (1/2) g(1/2) = 0 0 = 0 and
(f g)(1/3) = f (1/3) g(1/3) = 0 0 = 0. So, f g I. Since 0 I, I is an additive subgroup of R by
the subgroup criterion. If h R, then (f h)(1/2) = f (1/2)h(1/2) = 0 h(1/2) = 0 and
(f h)(1/2) = (hf )(1/2) = (f h)(1/3) = (hf )(1/3) = 0. So we have shown that I is an ideal of R.
I is not a prime ideal. If f (x) = x2 1/4 and g(x) = x2 1/9, (f g)(1/2) = (f g)(1/3) = 0 then f g I.
However, f
/ I and g
/ I since f (1/3) 6= 0 and g(1/2) 6= 0. //
Dummit & Foote Text Exercise 7.4-10: Assume R is commutative. Prove that if P is a prime ideal of
R and P contains no zero divisors then R is an integral domain.

MAT7400 Assignment #3

21

Solution: Let x, y R such that xy = 0. Since P is a prime ideal and xy P , x P . The following
equally valid if y is chosen instead of x. If x 6= 0 then since P contains no zero divisors in R, y = 0, //
Dummit & Foote Text Exercise 7.4-15: Let x2 + x + 1 be an element of the polynomial ring
E = F2 [x] and use the bar notation to denote passage to the quotient ring F2 [x]/(x2 + x + 1).
(a) Prove that E has 4 elements: 0, 1, x, and x + 1.
(b) Write out the 4 4 addition table for E and deduce that the additive group E is isomorphic to the
Klein 4-group.

(c) Write out the 4 4 multiplication table for E and prove that E is isomorphic to the cyclic group of
order 3. Deduce that E is a field.
Solution: Note: This problem is identical to Example 4.3.4 in Beachy & Blairs text, pages 208-209.
(a) The congruence classes modulo x2 + x + 1 are polynomials that that have degree less than 2 over F2 .
The congruence classes are 0, 1, x, and 1 + x.
(b)
0
1
x
1+x
+
0

1+x

1+x

1+x

1+x

1+x

Since the group has order four and each element except the identity has order 2, this group is isomorphic
to the Klein four group.
(c)
0
1
x 1+x

x 1+x

1+x

1+x

1+x

E = {1, x, 1 + x}. Since the order of E is three, its isomorphic to a cyclic group of order three.
(d) x2 + x + 1 is irreducible over the field F2 since it has no roots in F2 . Therefore F2 [x]/(x2 + x + 1) is a
field.//
Dummit & Foote Text Exercise 7.3-16: Let f (x) = x4 16 be an element of E = Z[x] and use the bar
notation to to denote passage to the quotient ring row Z[x]/(x4 16). (a) Find a polynomial of degree 3
which is congruent to g(x) = 7x13 11x9 + 5x5 2x3 + 3 modulo x4 16.
(b) Prove that x + 2 and x 2 are zero divisors in E.
Solution: (a). Using the division algorithm (and Matlab), we obtain the following:
g(x) = (7 x9 + 101 x5 + 1621 x) (x4 16) + (2 x3 + 25936 x + 3)
and thus, 2 x3 + 25936 x + 3 is the desired polynomial.
(b)x + 2 and x 2 are zero divisors since they are factors of x4 16. That is,
x4 16 = (x 2)(x + 2)(x2 + 4) implies 0 (x 2)(x + 2)(x2 + 4) mod x4 16.

MAT7400 Assignment #3

22

Dummit & Foote Text Exercise 7.4-18: Prove that if R is an integral domain and R[[x]] is the ring of
formal power series in the indeterminate x then the principal ideal generated by x is a prime ideal (cf.
Exercise 3, Section 2). Prove that the principal ideal generated by x is a maximal ideal if and only if R is a
field.
Solution We approach this problem in the same manner as Exercise 7.4-7.
P
If R is a commutative ring then : R[[x]] R defined by
ri xi 7= r0 is P
an surjective ring P
homomorphism and ker = (x). It is clear that is surjective. Let p(x) = ai xi and q(x) = bi xi . We
then have
X

(p + q) =
(ai + bi )xi = a0 + b0 = (p) + (q)
and

X X
(pq) = (
ai bj )xk = a0 b0 = (p)(q)
k

i+j=k

which show that is a ring homomorphism. Not concering ourselves regarding convergence makes me a bit
worried that the above is not entirely true.
P
ri xi ker .P
Then r0 = 0 (using the definition that
Now to show P
ker = (x).P
(ker (x)) Assume
0
i
i
0 = 1), and
ri x = x ri+1 x (x). ((x) ker) If y = x ai xi , then y (x) and that implies
(y) = 0, so that y ker .
By the First Isomorphism Theorem for rings, we have R[[x]]/ker = R[[x]]/(x)
= R. Proposition 13 tells
us that if R[[x]] is commutative, then the ideal (x) is a prime ideal in R[[x][ if and only if the quotient ring
R[[x]]/(x) is an integral domain. Since R[[x]]/(x)
= R, R is an integral domain if and only if
R[[x]]/(x)
= R is an integral domain.
Proposition 12 tells us that if R[[x]] is commutative, then the ideal (x) is maximal if and only if the
quotient ring R[[x]]/(x) is a field. We have shown by the first isomorphism theorem that R[[x]]/(x)
= R.
So R is a field if and only if R][x]]/(x) is a field.//
Dummit & Foote Text Exercise 7.4-19: Let R be a finite commutative ring with identity. Prove that
every prime ideal of R is a maximal ideal.
Solution: If P is a prime ideal of a ring R that is commutative and contains 1, then R/P is an integral
domain. Suppose that P is prime, and let a + P and b + P be elements of R/P . Suppose that their
product ab + P = 0 + P , the zero element of R/P . This implies ab0 P or equivalently ab P . Since P is
prime, either a P or b P . If a P , then a + P = 0 + P and if b P , then b + P = 0 + P . Either way,
the only way a product equals zero is when one of the factors equals zero, which along with R/P is
commutative implies R/P is an integral domain.
If R is a finite integral domain, then R is a field. Prof. Robert Ash of UIUC had a very simple proof this.
If a R and a 6= 0,the map x ax, x R,is injective because R is an integral domain (no zero divisors).
If R is finite,the map is surjective as well, so that ax = 1 for some x. Hence R is a field.
Since R/P is a finite integral domain, it is a field and therefore P R is maximal.//
Dummit & Foote Text Exercise 7.4-26: Prove that a prime ideal in a commutative ring R contains
every nilpotent element (cf. Exercise 13, Section 1). Deduce that the nilradical of R (cf. Exercise 29,
Section 3) is contained in the intersection of all the prime ideals of R. (It is shown in Section 15.2 that the
nilradical of R is equal to the intersection of all prime ideals of R.)
Solution: Let P R be a prime ideal and let r R be nilpotent with rn = 0. Let 1 m n be minimal
such that rm P . R/P is an integral domain and xm + P = 0 R/P . If m 2, we have
(r + P )(rm1 + P ) = 0, so that r + P is a zero divisor in R/P , a contradiction since R/P is an integral
domain. Therefore
T m = 1, and r P . Hence N(R) P . If P(R) is the collection of all prime ideals of R,
we have N(R) P(R).

MAT7400 Assignment #3

23

Dummit & Foote Text Exercise 7.4-28: Prove that if R is a commutative ring and
N = (a1 , a2 , . . . , an ) where each ai is a nilpotent element, then N is a nilpotent ideal (cf. Exercise 37,
Section 3). Deduce that if the nilradical of R is finitely generated then it is a nilpotent ideal.
Solution: First we complete Exercise 7.4-12 as we will need the result from it. Assume R is commutative
and suppose I = (a1 , a2 , . . . , an ) and J = (b1 , b2 . . . bm ) are two finitely generated ideals in R. Prove that
the product ideal IJ is finitely generated by the elements ai bj for i = 1, 2, . . . , n and j = 1.2, . . . , m.
Proof: Let H be the set containing the elements ai bj for i = 1, 2, . . . , n and j = 1.2, . . . , m.
P
P
P
(IJ
where
i =
j ti,j aj and si =
k ui,k bk . Then since R is commutative,
PH)
PLet x = Pi ri si IJ, P
P rP
x = i ( j ti,j aj )( k ui,k bk ) = i j k ti,j ui,k aj bk (H).
P
(IJ H) Let x =
ri,j ai bj (H). Since I is an ideal, x IJ.
P
i
= 0 for all ai A for some positive integer mi . M = mi . From
Back to the given problem. Assume am
i
P
Q
ki = M }. So for each
Exercise 7.4-12 (which we just completed), we have N M = X where X = { aki i |
Q ki
M
element ai X, some ki is at least mi and therefore X = 0 and that implies N = 0. Thus N is a
nilpotent ideal.
If N(R) is finitely generated, then each generator is (by definition) nilpotent. Thus N(R) is a nilpotent
ideal.//
Dummit & Foote Text Exercise 7.4-30: Let I be an ideal of the commutative ring R and define
radI = {r R | rn I for some n Z+ }
called the radical of I. Prove that radI is an ideal containing I and that (radI)/I = N(R/I) (cf. Exercise
29, Section 3).
Solution: For all a I, a1 I, so I radI. Hence radI is nonempty. Let a, b radI with an , bm I. The
binomial theorem tells us that
m+n
X m + n
ak bm+nk .
(a + b)m+n =
k
k=0

Since I is an ideal, if k n, then ak I, and if k < n, then bm+nk I. Since R is commutative, this
implies every term of (a + b)n+m I, and consequently (a + b)m+n I since I is a group under addition.
Next, for all r R, (ra)n = rn an I, and (a)n R. Hence radI is an ideal of R. //
x + I radI/I x radI xn I for some n 1 xn + I = 0 R/I for some n 1 (x + I)n = 0
R/I for some n 1 x + I N(R/I). Hence radI/I = N(R/I). //
Dummit & Foote Text Exercise 7.4-31: An ideal I of the commutative ring R is called a radical ideal
if radI = I .
(a) Prove that every prime ideal of R is a radical ideal.
(b) Let n > 1 be an integer. Prove that 0 is a radical ideal in Z/nZ is radical if and only if n is a product
of distince primes to the first power (i.e., n is squarefree). Deduce that (n) is a radical ideal in Z if and
only if n the product of distinct primes in Z.
Solution: (a) Let P R be a prime ideal. Assume x radP . Then for some n 1, xn P . Assume that
n is minimal with this property. If n 2, then xxn1 P . Since P is prime, then either x P or
xn1 P which contradicts our assumption that n is minimal. Therefore n = 1 and x P .
Q
(b) () Let the prime factorization
of n be n = pki i . Assume
that 0 is a radical ideal in Z/nZ, and
Q
Q
assume some ki 2. Then pi is nonzero in Z/nZ, and ( pi )max ki = 0, a contradiction. This implies n
is squarefree.
() Assume n is squarefree and let a Z/nZ with ak = 0 for some positive integer k. If some prime p
divides n but not a, then no power of a can be divisible by n, a contradiction since ak 0 mod n and p
divides n. Hence n divides a, and rad 0 = 0.

MAT7400 Assignment #3

24

I had help with this part of the problem. Let R be a commutative ring , let I R be an ideal, and let
J R be an ideal containing I. Then J/I is radical ideal in R/I if and only if J is a radical ideal in R.
Proof: ( Assume J/I is a radical ideal in R/I, and let x rad J. Then xm J for some m 1. Next
(x + I)m = xm + I J/I, so x + I rad J/I = J/I. Thus x J, and J is a radical ideal in R.
() Assume J is radical in R and let x + I rad J/I. Then (x + I)m = xm + I J/I for some m 1, so
that xm J. Thus x rad J = J, and x + I J/I Hence J/I is a radical ideal.
If (n) is an ideal of Z, then (n) is a radical ideal in Z if and only if 0 is a radical ideal in Z/(n) if and only
if n is square free. //
Dummit & Foote Text Exercise 7.4-33: Let R be the ring of all continuous functions from the closed
interval [0, 1] to R and for each c [0, 1] let Mc = {f R | f (c) = 0} (recall that Mc was shown to be a
maximal ideal of R).
(a) Prove that if M is any maximal ideal of R then there is a real number c [0, 1] such that M = Mc .
(b) Prove that if b and c are distinct points in [0, 1] then Mb 6= Mc .
(c) Prove that Mc is not equal to the principal ideal generated by x c.
(d) Prove that Mc is not a finitely generated ideal.
Solution: I had a lot of help on this problem. It was good to apply material from MAT5600.
(a) Proof by contradiction. Let M R be a maximal ideal and assume M 6= Mc c [0, 1]. Then for all
such c, there exists a function fc M such that fc (c) 6= 0. Assume that fc (c) > 0. Since fc is continuous,
there exists a positive real number c > 0 such that fc [(c c , c + c )] = 0. The set
{(c c , c + c ) [0, 1] | c [0, 1]}covers[0, 1]. Since [0, 1] is compact (closed and bounded subset of R, this
cover has a finite subcover by the definition of compact. Let K [0, 1] be the finite subcover K is finite
and {(c c , c + c ) [0, 1] | c K} covers [0, 1]. For each c K, define uc (x) = 1 + (x c)/c if
x (c c , c) [0, 1], uc (x) = 1 + (c x)/c if x [c, c + c )P
[0, 1], and uc (x) = 0 otherwise. Then uc R
and uc is zero outside of (c c , c + c ). Next consider g =
uc fc . Since fc M , we have g M .
However, for all x [0, 1], g(x) is positive since each uc (x)fc (x) is nonnegative and some uc (x)fc (x) is
positive. Thus g(x) > 0 for all x, and therefore 1/g R. This implies that M contains a unit, a
contradiction. Thus M = Mc for some c [0, 1].
(b) Assume b 6= c. Then x b Mb but (x b)(c) = c b 6= 0. This implies x b
/ Mc . Therefore,
Mb 6= Mc .
|x c|
. As
(c) Assume Mc = (x c). Then |x c| = f (x)(x c) for some f (x) R. If x 6= c, then f (x) =
xc
x approaches c from the right, f (x) approaches , while f(x) approaches as x approaches c from the
left. So, no extension of f is in R. Therefore, Mc is not generated by x c.
(d) Proof
by contradiction.
Assume Mc = (A) isfinitely generated,

withPA = {ai (x) | 1 i n}. Let


P
f=
|ai |. Then f is continuous
on
[0,
1]
and
f

M
.
Thus
f = P ri ai for some continuous
c

P
P
functions ri R. If we let r =
|ri |, we have f (x) = ri (x)ai (x)
|ri (x)||ai (x)| r(x)f (x). For
each b 6= c, there must exist a function ai such that
a
(b)
=
6
0,
otherwise
h(b)
= 0 for all h Mp
i
c and

x c Mc . Thus c is the only zero of f . From


f
(x)

r(x)f
(x),
for
x
=
6
c
we
have
r(x)

1/
f (x). As x
p
approaches c, f (x) approaches 0, so that 1/ f (x) is unbounded and that implies r(c) does not exist which
is a contradiction since r(x) R. Thus Mc is not finitely generated.
Dummit & Foote Text Exercise 7.4-37: A commutative ring R is called a local ring if it has a unique
maximal ideal. Prove that if R is a local ring with maximal ideal M then every element of R M is a unit.
Prove conversely that if R is a commutative ring with 1 in which the set of nonunits forms an ideal M ,
then R is a local ring with unique maximal ideal M .
Solution: Let x R M . If the ideal (x) is proper then it must be contained in some maximal ideal, and
M is the only maximal ideal. Thus (x) M , a contradiction since x
/ M . Thus (x) = R, and for some
y R we have xy = yx = 1. This implies x is a unit.
Assume the set M of nonunits in R form an ideal. To show that M is maximal, assume M ( I for some
ideal I. Then I contains a unit, so that I = R. To show that M is the unique maximal ideal, assume there

MAT7400 Assignment #3

25

is an ideal N R such that N 6 M . Then N contains some element x not in M , which is a unit. Thus
N = R. In particular, every proper ideal of R is contained in M . Thus M is the unique proper ideal of R,
and R is a local ring.
Dummit & Foote Text Exercise 7.4-38: Prove that the ring of all rational numbers whose
denominators is odd is a local ring whose unique maximal ideal is the principal ideal generated by 2.
Solution: R is a commutative ring with 1 6= 0. Note that 1 = 11 . Let M R be the set of nonunits. Since
R Q, every element a/b R has an inverse in Q, b/a. If a/b is invertible in R then its inverse is b/a.
However, a/b is not invertible in R when a is even (including zero). Thus M = {2a/b | a, b Z, b 6= 0}. So,
M (2). Since 2 itself is a nonunit in R, M = (2), that is, M is an ideal. By the previous exercise
(7.4-37), R is local with maximal ideal M = (2).
Dummit & Foote Text Exercise 7.4-39: Following the notation of Exercise 26 in Section 1, let K be a
field, let v be a discrete valuation on K and let R be the valuation ring of v. For each integer k 0 define
Ak = {r R | v(r) k} {0}.
(a) Prove that Ak is a principal ideal and that A0 A1 A2 . (b) Prove that if I is any nonzero
ideal of R, then I = Ak for some k 0. Deduce that R is a local ring with unique maximal ideal A1 .
Solution: (a) It is clear that Ak+1 Ak for all k. Claim: Ak is an ideal. Since 0 Ak , Ak is nonempty.
Let a, b Ak . If one or both of a and b is 0, then a + b Ak . If a + b = 0, then a + b Ak . If a + b 6= 0, we
have v(a + b) min(v(a), v(b)) k, so that a + b Ak . Next, v(a) = v(a) and that implies a Ak . If
r R then v(r) 0 and v(ra) = v(r) + v(a) k, so that ra Ak . Since K is commutative, Ak is an ideal
of R.
To show that Ak is principal, choose x Ak such that v(x) = k. We can choose one because that element
exists since v is surjective. Note that x1 exists in K, and that v(x1 ) = v(x). Let r Ak . Then v(r) k
and v(x1 r) = v(x1 ) + v(r) = v(r) v(x) 0. That implies x1 r R and r = xx1 r. Thus r (x), and
we have Ak (x), and thus Ak = (x). In particular, Ak is generated by any element of valuation k.
(b) Let I Ak be a nonzero ideal of R. Let k be minimal among v(r) for r I. Let a I such that
v(a) = k. In particular, we have I Ak . Moreover, since Ak = (a), we have Ak I. Hence I = Ak . From
part (a), every proper ideal of R is contained in A1 and therefore A1 is the unique maximal ideal of R.
Dummit & Foote Text Exercise 7.4-40: Assume R is commutative. Prove that the following are
equivalent: (see also Exercises 13 and 14 in Section 1)
(i) R has exactly one prime ideal
(ii) every element of R is either nilpotent or a unit
(iii) R/(R) is a field (cf. Exercise 29, Section 3).
Solution: (i) (ii) Assume R has exactly one prime ideal. Since every maximal ideal of R is prime, R is
local. By Exercise 7.4-26, N(R) is the unique maximal ideal. By Exercise 7.4-37, every element of
R N(R) is a unit, and (by definition) the remaining elements are nilpotent.
(ii) (iii) Assume x + N(R) is nonzero. Then x
/ N(R). Since x is not nilpotent in R, x is a unit. Then
x1 exists in R, and we have (x + N(R))(x1 + N(R)) = 1. Thus every nonzero element of R/N(R) is a
unit. Since R is commutative, R/N(R) is a field.
(iii) (i) Assume R/N(R) is a field. By Exercise 7.4-26, N(R) is contained in every prime ideal of R. By
the Lattice Isomorphism Theorem for rings, the only possible proper prime ideal is N(R). Also, N(R) is
prime since it is maximal in R. Thus N(R) is the unique prime ideal of R.
Dummit & Foote Text Exercise 7.5-3: Let F be a field. Prove that F contains a unique smallest
subfield F0 and that F0 is isomorphic to either Q or Z/pZ for some prime p (F0 is called the prime subfield
of F ). [See Exercise 26, Section 3.]

MAT7400 Assignment #3

26

Solution: We need to make use of the following fact and we will prove it:
T
T
Let F be a field and let X be a set of subfields of F . Then X is a subfield of F . Proof: X is a subring
of F , contains 1 since 1 E for all E X . Let r F . Then r E for each E X , and so r1 E. Then
r1 F , and hence F is a field. //
T
Let F be a field, and let F denote the set of all subfields of F . Then F0 = F is a subfield of F and is
contained in every other subfield.
Consider the ring homomorphism : Z F0 with (1) = 1. The induced map : Z/(n) F0 is an
injective ring homomorphism, Exercises 7.4-26 and 7.4-28 informs us that n is a prime or 0. For all nonzero
a Z/(n), (a) is a unit in F0 . Thus we have an monomorphism (injective
homomorphism) : F (Z/(n)) F0 , where F (Z/(n)) denotes the field of fractions. Note that im is a
subring of F which is a field, and thus im = F0 . Thus is an isomorphism. If n = 0, then F0
= Q, and if
n = p is prime, then F0
= Z/(p).//
Dummit & Foote Text Exercise 7.5-4: Prove that any sub field of R must contain Q.
Solution: Exercise 7.5-3 showed us that , R contains a unique inclusion-smallest subfield which is
isomorphic either to Z/(p) for a prime p or to Q.
Assume that the unique smallest subfield of R is isomorphic to Z/(p), and let a in this subfield be nonzero.
Then pa = 0 R, and since p R is a unit implies a = 0, which is a contradiction. Hence the unique
smallest subfield of R is isomorphic to Q. Any subfield of R contains a subfield isomorphic to Q.
In Section 10.1s exercises R is a ring with 1 and M is a left R-module.
Dummit & Foote Text Exercise 10.1-2: Prove that Rx and M satisfy the two axioms in Section 1.7
for a group action of the multiplicative group Rx on the set M .
Solution: R is a group under the multiplication. The restriction of the group action to R M is a
mapping R M M . Since R is a multiplicative group, it contains the multiplicative identity, 1. For all
m M , the second axiom is satisfied, 1 m = m. For all r1 , r2 R , (r1 r2 ) m = r1 (r2 m) and thereby
satisfies the first axiom.//
Dummit & Foote Text Exercise 10.1-5: For any left ideal I of R define
X
IM = {
ai mi | ai I, mi M
finite

to be the collection of all finite elements of the form am where a I and m M . Prove that IM is a
submodule of M .
Solution:
Apply
the definition of the submodule criterion! Since 0 0 = 0 IM , IM is not empty. Let
P
P
a

m
,
b

nk IM andP
letr R. Then
i
i
k
i
k
P
P
P
( i ai mi ) + r ( k bk nk ) = i ai mi + k (rbi ) nk IM . Hence IM M is a submodule. //
Dummit & Foote Text Exercise 10.1-15: If M is a finite abelian group then M is naturally a
Z-module. Can this action be extended to make M into a Q-module?.
Solution: Proof by contradiction. Let M be a (multiplicative) finite abelian group of order k. Assume
that M is a Q-module and that the action of Q on M extends the natural action of Z given by k m = mk .
Let x M be any nonidentity (and non-zero) element. Then k1 x = y for some y M . That leads to
k ( k1 x) = k y. So x = y k = 1, a contradiction. Therefore no such module structure exists.
Dummit & Foote Text Exercise 10.1-18: Let F = R, let V = R2 and let T be the linear
transformation from V to V which is rotation clockwise about the origin by /2 radians. Show that V and
0 are the only F [x]-submodules for this T .

MAT7400 Assignment #3

27

Solution: I had help on this problem as I didnt quite understand fully F [x] modules. Note that
T : V V is given by (x, y) 7 (y, x). Let N be an F [x]-submodule of V . Assume that there exists a
nonzero element (a, b) N . This implies either a 6= 0 or b 6= 0 and therefore a2 + b2 > 0. Let (x, y) V ,
bx ay
ax + by
, and r2 = 2
. Then
r1 = 2
a + b2
a + b2
(r1 + r2 x) (a, b)

= r1 (a, b) + r2 (x (a, b))


= (r1 a, r1 b) + (r2 b, r2 a)
= (r1 a + r2 b, r1 b r2 a)
= (r1 a + r2 b, r1 b r2 a)


axa + bya bxb ayb axb + byb bxa aya


+ 2
, 2
2
a 2 + b2
a + b2
a + b2
a + b2


=

a2 x + b2 x b2 y + a2 y
, 2
a2 + b2
a + b2

 2

a + b2 b2 + a2
x 2
,y
a + b2 a2 + b2

(x, y).

Hence (x, y) N and N = V since N V . So, every nonzero F [x]-submodule of V is V . Therefore, 0


andV arethe only F [x]-submodules of V .
Dummit & Foote Text Exercise 10.1-19: Let F = R, let V = R2 and let T be the linear
transformation from V to V which is projection onto the y-axis. Show that V , 0, the x-axis, and the y-axis
are the only F [x]-submodules for this T .
Solution: Let N V be an F [x] submodule. Let the x-axis be represented by X = R 0 and the
y axis be represented as Y = 0 R.
Assume there exists an element (a, b) N where a, b 6= 0 and let (x, y) V . Then
( xa + ( yb xa )x) (a, b) = (x, y). Thus V N , and since N V , that implies N = V .
Next assume either a = 0 or b = 0 for every element (a, b) N and assume that (a, 0) N with a 6= 0 and
there exists (0, b) N with b 6= 0. Then (a, b) N with a, b 6= 0, a contradiction. Thus N X. Now let
(x, 0) X. Since xa (a, 0) = (x, 0), X N . Hence N = X.
Next assume (0, b) N with b 6= 0. As before, N Y . If (0, y) Y , then yb (0, b) = (0, y), and so Y N .
Thus N = Y .
Last, if for all (a, b) N a = b = 0, then N = 0.
So, the only possible submodules of V are 0, X, Y , and V . 0 and V are trivial submodules. So, we need to
verify that X and Y are submodules by the submodule criterion.
Sincee (0, 0) X, X is nonempty. Let (a, 0), (b, 0) X, p(x) F [x], and p(x) = p0 + xp0 (x). Then
x (b, 0) = (0, 0) and
(a, 0) + p(x) (b, 0)

=
= (a, 0) + p0 (b, 0) + p0 (x) (x (b, 0))
= (a, 0) + (p0 b, 0) + (0, 0)
= (a + p0 b, 0) X

By the submodule criterion, X V is a submodule.


We repeat
P the process for Y . Since (0, 0) Y , Y is nonempty. Let (0, a), (0, b) X , p(x) F [x] and
p(x) = pi xi . Then T 2 (v) = T (v). In particular, xk v = x v for all k 1. If v Y , then T (v) = v. We
obtain the following:

MAT7400 Assignment #3

28

(0, a) + p(x) (0, b)

P
= (0, a) + (P pi xi ) (0, b)
= (0, a) + P pi (xi (0, b))
= (0, a) +P pi (0, b)
= (0, a + pi b) Y.

So, Y V is a submodule.
Dummit & Foote Text Exercise 10.2-1 : Use the submodule criterion to show that kernels and images
of R-module homomorphisms are submodules.
Solution: Let R be a ring with 1 and let M and N be left R-modules, and let : M N be an R-module
homomorphism. (0) = 0 implies that ker and im are both nonempty.
Let a, b ker and let r R. Then (a + r b) = (a) + r (b) = 0, so that a + r b ker . Hence ker
is an R-submodule of M by the submodule criterion.
Let x, y M and let r R. Then (x) + r (y) = (x + r y). Since (x) and (y) are arbitrary in im,
im is an R-submodule of N by the Submodule criterion.
Dummit & Foote Text Exercise 10.2-2 : Show that the relation is R-module isomorphic to is an
equivalence relation on any set of R-modules.
Solution: M
= N if there exists an R-module isomorphism : M N . To prove an equivalence relation,
we need to show that the relation is reflexive, symmetric, and transitive. Let S be a set of left R-modules.
(Reflexive). Let M S. The identity mapping id : M M is an R-module isomorphism since
id(x + r y) = x + r y = id(x) + r id(y) for all x, y M and r R. Henee M
= M , and so the relation
=
is reflexive.
(Symmetric). Assume that M, N S and that M
= N . This implies that there exists an Rmodule
homomorphism from : M N . Since is a bijection, has in inverse that is well defined. Let r R
and x, y N . Let x = (a) and y = (b) for some a, b M . Then
1 (x + r y)

=
=
=
=

1 ((a) + r (b))
1 ((a + r b))
a+rb
1 (x) + r 1 (y).

So 1 : N M is an R-module isomorphism. Hence N


= M and the relation is symmetric.

(Transitive). Let M, N, P S and M = N and N = P . Then there exist R-module isomorphisms


: M N and : N P . We know that : M T is a bijection (since function composition of
bijections is a bijection). To show that it is also an R-module homomorphism, let x, y M and r R.
Then
( )(x + r y)

= ((x + r y))
= ((x) + r (y))
= ((x)) + r ((y))
= ( )(x) + r ( )(y)
P . Hence the relation is transitive.
So : M P is an R-module isomorphism, and we have M =
Since all three requirement of an equivalence relation have been met, is R-module isomorphic to is an
equivalence relation.
Dummit & Foote Text Exercise 10.2-3: Give an explicit example of a map from one R-module to
another which is a group homomorphism but not an R-module homomorphism.
Solution: Let R be a non-commutative ring with 1. Choose an element a not in the center of R and then
consider R as a left module over itself via multiplication. That is, M = R. If (x) = a x defines the map

MAT7400 Assignment #3

29

: M M , we have a group homomorphism (an endomorphism) since


(x + y) = a (x + y) = a x + a y = (x) + (y). However, if b R does not commute with a, then
(b) = a b = ab but b (1) = b (a 1) = b a = ba 6= ab. Hence , although a group homomorphism, is not
an R-module homorphism.
Dummit & Foote Text Exercise 10.2-13: Let I be a nilpotent ideal in a commutative ring R (cf.
Exercise 37, Section 7.3), let M and N be R-modules and let : M N be an R-module homomorphism.
Show that if the induced map : M/IM N/IN is surjective, then is surjective.
Solution: The induced map is given by (m + IM ) = (m) + IN .
Since is surjective, N/IN = [M/IM ] = ([M ] + IN )/IN . By the Lattice Isomorphism theorem for
modules, we have N = [M ] + IN .
We prove the statement N = [M ] + I t N for all t 1 by induction. The base case t = 1 has been shown to
be true. Assume for the induction step the equation holds for some t > 1. Then
N

=
=
=
=

[M ] + I t N
[M ] + I t ([M ] + IN )
[M ] + I t [M ] + I t+1 N
[M ] + I t+1 N since I t [M ] [M ].

The induction hypothesis is proved. Since I k = 0, N = [M ] + I k N = [M ] and implies is surjective.


Dummit & Foote Text Exercise 10.2-14: Let R = Z[x] be the ring of polynomials in x and let
A = Z[t1 , t2 , . . . , ] be the ring of polynomials in the independent indeterminates t1 , t2 , . . . . Define an action
of R on A as follows: 1) let 1 R act on A as the identity, 2) for n 1, let xn 1 = tn , let xn ti = tn+i
for i = 1, 2, . . . and let xn act as 0 on monomials in A of (total) degree at least two, and 3) extend
Z-linearly, i.e., so that the module axioms 2(a) and 2(c) are satisfied.
(a) Show that xp+q ti = xp (xq ti ) = tp+q+i and use this to show that under this action the ring A is a
(unital) R-module.
(b)Show that the map : R A defined by (r) = r 1A is an R-module homomorphism of the ring R
into the ring A mapping 1R to 1A , but is not a ring homomorphism from R to A.
Solution: (a) xp+q ti = tp+q+i = xp (tq+i ) = xp (xq ti ). Hence makes A into a unital left R-module.
(b) Let a, b, r R. Then
(a + r b) = (a + r b) 1
= a 1 + r (b 1)
= (a) + r (b)
so that is an R-module homomorphism.
To show its not a ring homomorphism, we use contradiction. Assume that is a ring homomorphism.
Then (x2 ) = x2 1 = t2 but (x2 ) = (x)(x) = (x 1)(x 1) = t1 2 contradiction. Hence is not a ring
homomorphism.
Dummit & Foote Text Exercise 10.3-1: Prove that if A and B are sets of the same cardinality, then
the free modules F (A) and F (B) are isomorphic.
Solution: I had help on this problem. Prof. R.D. Maddux of Iowa State University had the most lucid
explanation for this problem: http://orion.math.iastate.edu/maddux/505-Spring-2010/hw05.3.pdf
Since A and B have the same cardinality, there exists a bijection f : A B. Since f is a bijection, its
inverse is also a bijection, that is, f 1 : B A. A. By Theorem 10.6 the free modules F (A) and F (B)
have the universal mapping property. We now apply this property to f . Note that the image of f is B
since f is surjective, and B is a subset of the free module F (B). Thus we have another function that maps
A into F (B) and is equal to f on all elements of A, that is, g : A F (B) and g(a) = f (a) for all a A.

MAT7400 Assignment #3

30

By the universal mapping property there is a unique R-module homomorphism : F (A) F (B) which
extends g (and f ). That is, f (a) = g(a) = (a) for all a = inA. Using the same reasoning, with A and B
interchanged and f replaced by f 1 , we obtain another R-module homomorphism : F (B) F (A)
extending f 1 . The composition of R-module homomorphisms is again an R-module homomorphism:
: F (A) F (A). For every a A, ( )(a) = ((a)) = (f (a). Since f (a) B and extends f 1 ,
(f (a)) = f 1 (f (a)) = a. This shows that is an extension of the identity map : A F (A) which
sends every element of A to itself (i.e, (a) = a for every a A. By the universal mapping property, has a
unique extension to an R-module homomorphism from F (A) to itself. The identity map from F (A) to
F (A) is an R-module homomorphism of F (A) onto itself, and by the universal mapping property of F (A)
it is the only R-module homomorphism of F (A) onto itself. However, is a homomorphism of F (A)
onto itself that extends the identiy map on A so must be the identity map from F (A) to F (A).
Similarly, is the identity map from F (B) to F (B). Thus and are R-module homomorphisms
between F (A) and F (B), and they are inverses of each other, so they are both injective and surjective, and
therefore are isomorphisms between F (A) and F (B). Thus, F (A)
= F (B) whenever |A| = |B|. //
Rm if and only
Dummit & Foote Text Exercise 10.3-2: Assume R is commutative. Prove that Rn =
if n = m, i.e., two free R-modules of finite rank are isomorphic if and only if they have the same rank.
[Apply Exercise 12 of Section 2 with I a maximal ideal of R. You may assume that if F is a field, then
Fn
= F m if and only if n = m, i.e., two finite dimensional vector spaces over F are isomorphic if and only
if they have the same dimension - this will be proved later in Section 11.1.]
Solution: There are two solutions. One easy and one a bit more involved.
Simpler way. Assume that Rn
= Rm . Let I be a maximal ideal of R. Then Rn /IRn
= Rm /IRm . Exercise
(R/IR)

(R/IR)

(R/IR)
(n
times).
This implies
10.2-12 (not assigned) tells that Rn /IRn
=R
m
(R/I)n
(R/I)
.
Since
these
are
vector
spaces
over
the
field
R/I,
we
have
n
=
m.
//
=
The more complicated way.
It is clear that if n = m, then the result holds. Assume hat Rm
= Rn , where R : Rm Rn is an
R-module isomorphism. Let I R be a maximal ideal. Next : Rm Rn /IRn , where denotes the
natural projection.
Claim: ker = IRm .
P
P
P
(IRm ker ): If x =
ai (ri,j ) IRm , then ( ai (ri,j )) = ai ((ri,j )) IRn . Thus
( )(x) = 0, and we have IRm ker .
(ker P
IRm ): Assume x = (ri ) ker . Then 0 = ( )(x) = (x) + IRn . Thus (x) IRn . Let
m
(x) = ai (ri,j
Rn . Since is surjective, we have (ri,j ) = ((s
P), where (ri,j ) P
Pi,j )) for some (si,j ) R .
Then (x) = eo ai ((si,j )) = ( ai (si,j )). Since is injective, we have x =
ai (si,j ), and thus
x IRm as desired. Certainly is also surjective.
By the first isomorphism theorem, the induced map : Rm /IRm Rn /IRn is an R-module isomorphism.
The result of Exercise 10.2-12 implies (as with the easier method) that (R/I)m
=R (R/I)n . Since I is
maximal, R/I is a field, and therefore m = n.//
Dummit & Foote Text Exercise 10.3-4: An R-module M is called a torsion module if for each m M
there is a nonzero element r R such that rm = 0, where r may depend on m (i.e., M = Tor(M ) in the
notation of Exercise 8 of Section 1). Prove that every finite abelian group is a torsion Zmodule. Give an
example of an infinite abelian group that is a torsion Zmodule.
Solution: Let M be a finite abelian group. This implies it is also a Z-module. Let n be the order of M .
Then for every a M , by Lagrange, the order of a divides the order of n of the the abelian group M .
Hence, na = 0. Since n Z + and na is the result of the action of n on a in the module M , we have
a Tor(M ), so M Tor(M ). Tor(M ) M is obviously true. Therefore M = Tor(M ). //
Q
Let N = iN Z/2Z and this implies that N is a direct product of countably many copies of the 2-element
cyclic group Z/2Z. But N is an infinite abelian group whose cardinality is the same as the set of real
numbers (by Cantors Diagonal Argument). Every element of N has order 2 so 2 n = 0 for every n N .
Hence Tor(N ) = N .

MAT7400 Assignment #3

31

Dummit & Foote Text Exercise 10.3-6 Prove that if M is a finitely generated R-module that is
generated by n elements then every quotient of M may be generated by n (or fewer) elements. Deduce that
quotients of cyclic modules are cyclic.
Solution: We need to show M/N = (ai + N | i {1, 2, 3 , n 1, n}).
((ai + N | i {1, 2, 3 , n 1, n}) M/N ) holds by the definition of a quotient.
P
(M/N ) P(ai + N | i {1, 2, 3 , n 1, n}). Assume x + N M/N where x = i ri ai . Then
x + n = ri (ai + N ) as required. Hence {ai + N }ni=1 is an R-module generating set for M/N .//
If M is cyclic, then it has a generating set consisting of a single element. Moreover, the quotient of M is
then generated by at most one element. Since every module is generated by at least one element, every
quotient of M is also cyclic.
Dummit & Foote Text Exercise 10.3-7: Let N be a submodule of M . Prove that if both M/N and N
are finitely generated then so is M .
Solution: If M/N = (ai + N | 1 i m) and N = (bj | 1 j n), then
M = (ai , bj | 1 i m, 1 j n).
P
P
M (ai , bjP| 1 i m, 1 j n)) Assume m P
M . ThenP
m + N = i ri (ai + N ) = ( i ri ai ) + N .
ThenP
m i ri P
ai N and that implies m i ri ai = j sj bj . This leads to
m = i ri ai + j sj bj (ai , bj | 1 i m, 1 j n). Hence M is finitely generated.//
(ai , bj | 1 i m, 1 j n) M ) holds.
Dummit & Foote Text Exercise 10.3-9: An R-module M is called irreducible if M 6= 0 and if 0 and
M are the only submodules of M . Show that M is irreducible if and only if M 6= 0 M is a cyclic module
with any nonzero element as generator. Determine all the irreducible Z-modules.
Solution: () If M is irreducible, then M 6= 0 by definition. Let x M be nonzero. Then Rx M is a
nonzero submodule since 1 x = x Rx. Since M is irreducible, Rx = M . Hence any nonzero element of
M generates M .
() Assume M 6= 0 and that if x 6= 0, then Rx = M . Let N M be a nonzero submodule. Then there
exists some nonzero x N , and M = Rx N . Thus N = M . Since the only submodules of M are 0 and
M , M is irreducible.//
We know that Z-modules are abelian groups, and that Z-submodules are the subgroups of abelian groups.
If M is an irreducible Z-module, it is a abelian group whose subgroups that has no nontrivial proper
subgroups). Hence M is cyclic and therefore it is of prime order.
Dummit & Foote Text Exercise 10.3-10: Assume R is commutative. Show that an R-module M is
irreducible if and only if M is isomorphic (as an R-module) to R/I where I is a maximal ideal of R. [By
the previous exercise, if M is irreducible there is a natural map R M defined by r 7 rm, where m is any
fixed nonzero element of M .]
Solution: () Assume M is irreducible, let m M a fixed element and be nonzero, and define
m : R M bym (r) = r m. Then for all x, y R and r R,
m (x + r y) = (x + r y) m = x m + r (y m) = m (x) + r m (y). So m is an R-module
homomorphism. Since m 6= 0 and M is irreducible, by Exercise 10.3.9, M = Rm. If b M , then there
exists a R such that b = a m = m (a). Hence m is surjective.
We need to show that ker m is a maximal ideal. Let x + ker m be nonzero. Then x m 6= 0. Since M is
irreducible, M = R(x m). This implies m = y (x m) = (yx) m for some y R. Then
1 m (yx) m = 0, so that 1 yx ker m . That is, (y + ker m )(x + ker m ) = 1 + ker m . So every
nonzero element of R/ker m has a left inverse. Because R/ker m is commutative, R/ker m is a field, so
that ker m is a maximal ideal of R.
R R/ker m , where ker m is a maximal ideal.
By the First Isomorphism Theorem, =

MAT7400 Assignment #3

32

() Assume I R is a maximal ideal. Then R/I is a field. Let x + I be nonzero. Then there exists y R
such that (y + I)(x + I) = (1 + I). If r + I R/I, then r + I = (ry + I)(x + I) which implies
R/I = R(x + I). That is, R/I is generated (as an R-module) by any nonzero element. By Exercise 10.3-9,
R/I is an irreducible R-module.
Dummit & Foote Text Exercise 10.3-11: Show that if M1 and M2 are irreducible R-modules, then
any nonzero R-module homomorphism from M1 to M2 is an isomorphism. Deduce that if M is irreducible
then EndR (M ) is a division ring (this result is called Schurs Lemma). [Consider the kernel and the image.]
Solution: Assume : M1 M2 is an R-module homomorphism with M1 and M2 irreducible. In Exercise
10.2-1, we proved that ker and im are submodules of M1 and M2 respectively. Since is not the zero
homomorphism, its kernel is not all of M1 , and since M1 is irreducible ker = 0. Thus is injective.
Similarly, im is not the zero submodule, and consequently M2 is not the zero submodule. Hence is
surjective. Therefore is an R-module isomorphism.
Let M be an irreducible unital left R-module. We know that EndR (M ) is a ring under pointwise addition
and composition. If EndR (M ) is nonzero, then by the first part of this exercise, it is an isomorphism
and so has an inverse 1 which is also in EndR (M ). So EndR (M ) is a division ring. We are not given any
additional information that would lead us to conclude that EndR (M ) is commutative and therefore a field.
Dummit & Foote Text Exercise 10.3-12: Let R be a commutative ring and let A, B, and M be
R-modules. Prove following isomorphisms of R-modules:
(a) HomR (A B, M )
= HomR (A, M ) HomR (B, M )
(b) HomR (M, A B)
= HomR (M, A) HomR (M, B).
Solution: I had A LOT of help with this problem.
(a) Let f : A M and g : B M , be R-module homomorphisms defined by (f, g) : A B M by
(f, g)(a, b) = f (a) + g(b). Then for all a1 , a2 A, b1 , b2 B, and r R, we have
(f, g)((a1 , b1 ) + r (a2 , b2 ))

=
=
=
=

(f, g)((a1 + r a2 , b1 + r b2 ))
f (a1 + r a2 ) + g(b1 + r b2 )
f (a1 ) + r f (a2 ) + g(b1 ) + r g(b2 )
(f, g)(a1 , b1 ) + r (f, g)(a2 , b2 ).

So (f, g) : A B M is an R-module homomorphism.


Now consider the map : HomR (A, M ) HomR (B, M ) HomR (A B, M ). We will show that is an
R-module homomorphism. Let f1 , f2 : A M and g1 , g2 : B M be R-module homomorphisms and let
r R. Then
((a1 , b1 ) + r (a2 , b2 ))(a, b) = (f1 + r f2 , g1 + r g2 )(a, b)
= (f1 + r f2 )(a) + (g1 + r g2 )(b)
= f1 (a) + r f2 (a) + g1 (b) + r g2 (b)
= (f1 , g1 )(a, b) + r (f2 , g2 )(a, b)
= ((f1 , g1 ) + r (f2 , g2 ))(a, b).
So is an R-module homomorphism. Next we show is an injection. Assume (f, g) ker . Then
0 = (f, g)(a, 0) = f (a) for all a A, so that f = 0. Similarly, g = 0. Thus (f, g) = 0 and ker = 0. Hence
is an injection.
Next we show is an surjection. Let : A B M be an R-module homomorphism, and define
f : A M and g : B M by f (a) = (a, 0) and g (b) = (0, b). Then
f (a1 + r a2 )

=
=
=

(a1 + r a2 , 0
(a1 , 0) + r (a2 , 0)
f (a1 ) + r f (a2 ).

MAT7400 Assignment #3

33

So f is an R-module homomorphism. Similarly, g is an R-module homomorphism. Since


(f , g )(a, b)

= f (a) + g (b)
= (a, 0) + (0, b)
= (a, b),

we have (f , g ) = . Hence is a surjection.


Thus, we have HomR (A B, M )
=R HomR (A, M ) HomR (B, M ). //
(b) Now let A : A B A and B : A B B be the left and right coordinate projections respectively
and these are R-module homomorphisms. Define : HomR (M, A B) HomR (M, A) HomR (M, B) by
() = (A , B ). If , HomR (M, A B) and r R, then
( + r )

= (A ( + r ), B ( + r ))
= (A + r (A ), B + r (B ))
= (A , A ) + r (B , B )
= () + r ().

Thus is an R-module homomorphism. We now show that is an injection. Assume ker . If


m M , then
0 = ()(m, m)
= ((A )(m), (b )(m))
= (A ((m)), B ((m)) = (m).
So = 0, hence ker = 0, and thus is an injection.
Next we show that P si is a surjection. Assume (f, g) HomR (M, A) HomR (M, B). Define
f,g : M A B by f,g (m) = (f (m), g(m). f,g is an R-module homomorphism. Moreover,
(f,g )(m) = ((A f,g )(m), (B f,g )(m)) = (f (m), g(m)). Thus (f,g ) = (f, g), and so is a
surjection . Hence HomR (M, A B)
=R HomR (M, A) HomR (M, B).//
Dummit & Foote Text Exercise 10.3-13: Let R be a commutative ring and let F be a free R-module
of finite rank. Prove the following isomorphism of R-modules: HomR (F, R)
= F.
n
Solution: Let F be a free R-module of finite
P rank. That is, let F be free on the set {ai }i=1 . Then every
element P
of F can be written uniquely as
ri ai for some ri R. Next, define : HomR (F, R) F by
() = (ai ) ai . is an R-module homomorphism. If , HomR (F, R) and r R, then

( + r )

P
= P( + r )(ai ) aiP
= ( (ai ) ai ) + r ( (ai ) ai )
= () + r ()

shows that is an R-module homomorphism.


P
is injective. Assume ker . Then 0 = () = (ai ) ai . Since F is free on the ai , we have
(ai ) = 0 for all ai , and thus = 0. So ker = 0, and thus is injective.
P
P
P
is surjective. Let
ti ai F . Define g : F R by
P g( ri ai ) = ri ti . So , g is an R-module
homomorphism. Moreover, g(ai ) = ti , so that (g) = ti ai . Hence is surjective.
Thus is an R-module isomorphism, so that we have HomR (F, R)
= F .//
Dummit & Foote Text Exercise 10.3-20: Let I be a no empty index set and for each i I let Mi be
an R-module. The direct product of the modules Mi is defined to be their direct product as abelian groups
(cf. Exercise 15 in Section 5.1) with the action of R componentwise multiplication. The direct sum of the
modules Mi is defined to be the restricted direct product of the abelian groups Mi (cf. Exercise 17 in
Section 5.1) with the action of R componentwise
multiplication. In other words,
the direct sum of the Mi s
Q
Q
is the subset of the direct product, iI Mi , which consists of all elements iI mi such that only finitely
many of the components mi are nonzero; the action of R on the direct product or direct sum is given by

MAT7400 Assignment #3

34

Q
Q
r iI mi = iI rmi (cf. Appendix
I for the definition of Cartesian products of infinitely many sets). The
L
direct sum will be denoted by iI Mi .
(a) Prove that the direct product of the Mi s is an R-module and the direct sum of the Mi s is a
submodule of their direct product.
(b) Show that if R = Z, I = Z+ and Mi is the cyclic group of order i for each i, then the direct sum of the
Mi s is not isomorphic to their direct product. [Look at torsion.]
Q
Solution: (a) We know from Exercise 5.1-15, that iI Mi is an abelian group under point-wise addition
and multiplication. So we just need
Q to verify the three left-module axioms.
Let r, s R and let (mi ), (ni ) iI Mi . Then
(r + s) (mi )

(rs) (mi )

r ((mi ) + (ni ))

= ((r + s) mi )
= (r mi + s mi )
= (r mi ) + (s mi )
= r (mi ) + s (mi ),
= ((rs) mi )
= (r (s mi ))
= r (s mi )
= r (s (mi )),
=
=
=
=
=

r (mi + ni )
(r (mi + ni ))
(r mi + r ni )
(r mi ) + (r ni )
r (mi ) + r (ni ).

Q
Thus iI MiQis a left R-module. If R has a 1 and each Mi is unital, then 1 (mi ) = (1 mi ) = (mi ), for all
(mi ), so that I Mi is unital.
L
Q
L
To show that iI M
we apply the submodule criterion.
iI Mi is
Li is a submodule of iI Mi ,L
nonempty since 0 iI Mi . Now let (xi ), (yi ) iI Mi and let r R. Let J, K I such that xj = 0
for j
/ J and yk = 0 for k
/ K. Consider L
(xi ) + r (yi ) = (xi + r yi ). If i
/ J L
K, then xi +
Qr yi = 0.
Since J K I is finite, (xi ) + r (yi ) iI Mi . By the submodule criterion, iI Mi iI Mi is an
R-submodule.
L
Q
(b) Consider the two Z-modules: M = N Z/nZ and N = N Z/nZ (theyre both abelian groups). The
goal is to show that these two modules are not isomorphic by verifying that one is torsion while the other is
not. Let (ai ) M . By definition of the direct sum, there is a natural number L such that, for all k L,
QL
ak = 0. Now let t = k=1 n . Then t (ai ) = 0 and that implies every element of M is torsion, so that M is
torsion. However, t (1) = (t) is nonzero for all integers t so N is not torsion. To put it another way, if
there existed r Z such that r(1, 1, 1, . . . ) = (0, 0, 0, . . . ), then we have a contradiction since
r 1 = r 6= 0 Z(n + 1)Z. Hence the direct product is not a torsion module. Thus M and N are not
isomorphic as Z-modules.

Dummit & Foote Text Exercise 10.3-23 : Show that any direct sum of free R-modules is free.
Solution: I had help on this problem. Let R be a ring with 1 and let {Fi }I be a family of free unital
Sleft
R-modules. Let Ai = {ai,j } be a free basis for each Fi and assume that the Ai are disjoint. Let A = I Ai .
Let F (A) denote the free R-module on A.
L
There is the natural inclusion A F (A) and a natural inclusion : A I Fi , where ai,j 7 (bt ), where
bt = ai,j if t = i and 0 otherwise.
L By the universal property of free modules, there is a unique R-module
homomorphism : F (A) I Fi such that (ai,j ) = (ai,j ). Since weve established that is a
R-module homomorphism, we show that is an R-module isomorphism.
P
P
is injective. Assume x ker and let x = ri,j ai,j . Then 0 = (x)i = j ri,j ai,j Fi . Since Fi is
free on Ai , ri,j = 0 for all i and j. This implies x = 0 and ker = 0. Hence is injective.

MAT7400 Assignment #3

35

P
L

ri,j ai,j ) I FP
i . Since only finitely many of the ri,j are nonzero,
P is surjective. Assume ( jP
r
a

F
(A).
Then
(
r
a
)
=
(
i,j i,j i,j
i,j i,j i,j
j ri,j ai,j ), so that is surjective.
L
L

Therefore I Fi =R F (A) and I Fi is free as an R-module.


Dummit & Foote Text Exercise 10.3-24: (An aribitrary direct product of free modules need not
Q be
free) For each positive integer I, let mi be the free Z-module Z, and let M be the direct product Z+ Mi
(cf. Exercise 20). Each element of M can be written uniquely in the form (a1 , a2 , a3 , . . . ) with ai Z for
all i. Let N be the submodule of M consisting of all such tuples with only finitely many nonzero ai .
Assume M is a free Z-module with basis B.
(a) Show that N is countable.
(b) Show that there is a countable subset B1 of B such that N is contained in the submodule N1 generated
by B1 . Show also that N1 is countable.
(c) Let M = M/N1 . Show that M is a free Z-module. Deduce that if x is any nonzero element of M then
there are only finitely many distinct positive integers k such that x = km some m M (depending on k).
(d) Let S = {(b1 , b2 , b3 , . . . ) | bi = i! for all i}. Prove that S is uncountable. Deduce that there exists
some s S such that s
/ N1 .
(e) Show that the assumption M is free leads to a contradiction: By (d) we may choose s S with s
/ N1 .
Show that for each positive integer k there is some m M with s = km, contrary to (c). [Use the fact that
N N1 .]
Solution: I had a lot of help on this problem. Especially part (3). Part (e) was the most difficult to
understand. (a) For each n Z+ , there is a natural inclusion n : Zn N given bySn (a)i = ai+1 if
0 i < n and 0 otherwise. Each element of N is in the image of some n . So, N = nN+ im n . Hence, N
is a countable union of countable sets, and therefore is countable.
P
S
(b) For each x N , there is a finite subset Bx B such that x = P
B1 = xN Bx . As the
bBx rb b. LetP
countable union
P of finite sets, B1 is countable. Since x N and x = bBx rb b bB1 Rb,
N N1 = bB1 Rb and N1 is also countable.
(c) The natural inclusion : B B1 M/N1 is given by b 7 b + N1 . By the universal property of free
modules, there is a unique R-module homomorphism : F (B B1 ) M/N1 such that (b) = b + N1 for
all b B B1 .
Now to show that is an isomorphism.
P
P
P
is injective. Let x ker P hi and let x =
ri bi . Then 0 =
ri bi + N1 , so that
ri bi N1 . Thus
ri = 0, and we have x = 0. So is injective.
P
is surjective. Assume xP+ N1 M/N1 . Since M is free on B, we have x + N1 =
ri bi + N1 for some ri
and bi B B1 . Then ( ri bi ) = x and that implies is surjective.
Therefore is an R-module isomorphism, and M/N1 is free as a Z-module.
Q
(d) There exists a bijection : S N {1, 1} = T . That is bi 7 +1 if bi > 0 and bi 7 1 if bi < 0. A
diagonalization argument (a modification of Cantors) shows that T is uncountable. Assume f : N T is a
bijection. Construct g = (ei ) T as follows: ei = 1 if f (i)i = 1 and 1 otherwise. Since g differs from
every element in im f = T in some component, g
/ im T , a contradiction. Thus S is uncountable. Since N1
is countable, there must exist s S such that s
/ N1 .
(e) Let k Z be positive. Define m M by mi = i! if i! < k and mi = i i!/k if i k, where i = 1 if
si > 0 and 1 if si < 0. Now (s km)i = 0 for all i > k, so that s km N N1 . Thus
s + N1 = k(m + N1 ). This is a contradiction of part (d) above. Thus M is not free as a Z-module.
Dummit & Foote Text Exercise 10.3-25: In the construction of direct limits, Exercise 8 of Section
7.6, show that if all Ai are R-modules and the maps ij are R-module homomorphisms, then the direct
limit A = lim Ai may be given the structure of an R-module in a natural way such that the maps

i : Ai A are all Rmodule homomorphisms. Verify the corresponding universal property (part (e)) for
R- module homomorphisms : Ai C commuting with the ij .

MAT7400 Assignment #3

36

Solution: ?
Dummit & Foote Text Exercise 10.3-26: Carry out the analysis of the preceding exercise
corresponding to inverse limits to show that an inverse limit of R-modules is an R-module satisfying the
appropriate universal property (cf. Exercise 10 of Section 7.6).
Solution: ?
Dummit & Foote Text Exercise 10.4-2: Show that the element 2 1 is 0 in Z Z Z/2Z but
non-zero in 2Z Z Z/2Z.
Solution:
Dummit & Foote Text Exercise 10.4-3: Show that C R C and C C C are both left R-modules but
are not isomorphic as R-modules.
Solution:
Dummit & Foote Text Exercise 10.4-4: Show that Q Z Q and Q Q Q are isomorphic left
Q-modules. [Show they are both 1-dimensional vector spaces over Q.]
Solution:
Dummit & Foote Text Exercise 10.4-6: If R is any integral domain with quotient field Q, prove that
(Q/R) R (Q/R) = 0.
Solution:
Dummit & Foote Text Exercise 10.4-13: Prove that the usual dot product of vectors defined by
letting (a1 , . . . , an ) (b1 , . . . , bn ) be a1 b1 + an bn is a bilinear map from Rn Rn to R.
Solution:
Dummit & Foote Text Exercise 10.4-14: Let I be an arbitrary nonempty index set and for each i I
let Ni be a left R-module. Let M be a right R-module. Prove the group isomorphism :
M (iI Ni )
= (M Ni ), where the direct sum of an arbitrary collection of modules is defined in
Exercise 20, Section 3. [Use the same argument as for the direct sum of two modules, taking care to note
where the direct sum hypothesis is needed- cf. the next exercise.]
Solution:
Dummit & Foote Text Exercise 10.4-15: Show that tensor products do not commute with direct
products in general. [Consider the extension of scalars from Z to Q of the direct product of the modules
Mi = Z/2i Z, i = 1, 2, . . . ].
Solution:
Dummit & Foote Text Exercise 10.4-18: Suppose I is a principal ideal in the integral domain R.
Prove that the R-module I R IEO has no nonzero torsion elements (i.e., rm = 0 with 0 6= r R and
m I R I implies that m = 0).
Solution:

MAT7400 Assignment #3

37

Dummit & Foote Text Exercise 10.4-19: Let I = (2, x) be the ideal generated by 2 and x in the ring
R = Z[x] as in Exercise 17. Show that the nonzero element 2 x x 2 in I R I is a torsion element.
Show in fact that 2 x x 2 is annihilated by both 2 and x and that the submodule of I R I generated
by 2 x x 2 is isomorphic to R/I.
Solution:
Dummit & Foote Text Exercise 10.4-21: Suppose R is commutative and let I and J be ideals of R.
(a) Show there is a surjective R-module homomorphism from I R J to the product ideal IJ mapping
i j to the element ij.
(b) Give an example to show that the map in (a) need not be injective (cf. Exercise 17).
Solution:
Dummit & Foote Text Exercise 10.4-24: Prove that the extension of scalars from Z to the Gaussian
integers Z[i] of the ring R is isomorphic to C as a ring: Z[i]]] Z R
= C as rings.
Solution:
Dummit & Foote Text Exercise 10.5-1: .
Solution:
Dummit & Foote Text Exercise 10.5-2: .
Solution:
Dummit & Foote Text Exercise 10.5-3: Let P1 and P2 be R-modules. Prove that P1 P2 is a
projective R-module if and only if both P1 and P2 are projective.
Solution:
Dummit & Foote Text Exercise 10.5-4: Let Q1 and Q2 be R-modules. Prove that P1 P2 is an
injective R-module if and only if both Q1 and Q2 are injective.
Solution:
Dummit & Foote Text Exercise 10.5-5: Let A1 and A2 be R-modules. Prove that A1 A2 is a P
flat
R-module if and only if both A1 and A2 are flat. More generally, prove that an arbitrary direct sum
Ai
of R-modules is flat if and only if each A is flat. [Use the fact that tensor product commutes with arbitrary
direct sums.]
Solution:
Dummit & Foote Text Exercise 10.5-6: Prove that the following are equivalent for a ring R:
(i) Every R-module is projective.
(ii) Every R-module is injective.
Solution:

MAT7400 Assignment #3

38

Dummit & Foote Text Exercise 10.5-7: Let A be a nonzero finite abelian group.
(a) Prove that A is not a projective Z-module.
(b) Prove that A is not an injective Z-module.
Solution:
Dummit & Foote Text Exercise 10.5-9: Assume R is commutative with 1.
(a) Prove that the tensor product of two free R-modules is free. [Use the fact that tensor products
commute with direct sums.] (b) Use (a) to prove that the tensor product of two projective R-modules is
projective.
Solution:
Dummit & Foote Text Exercise 10.5-12: Let A be an R-module, let I be any nonempty index set and
for each i I let Bi be an R-module. Prove the following isomorphisms of abelian groups; when R is
commutative prove also that these are R-module isomorphisms. (Arbitrary direct sums and direct products
of modules L
are introduced
Q in Exercise 20 of Section 3.) Q
Q
(a) HomR ( I Bi , A)
= I HomR (Bi , A) (b) HomR (A, I Bi )
= I HomR (A, Bi ).
Solution:
Dummit & Foote Text Exercise 10.5-14: Let

0LM N 0
be a sequence of R-modules.
(a) Prove that the associated sequence
0

0 HomR (D, L) HomR (D, M ) HomR (D, N ) 0


is a short exact sequence of abelian groups for all R-modules D if and only if the original sequence is a split
short exact sequence. [To show the sequence splits, take D = N and show the lift of the identity map in
HomR (N, N ) to HomR (N, M ) is a splitting homomorphism for .]
(b) Prove that the associated sequence
0

0 HomR (N, D) HomR (M, D) HomR (L, D) 0


is a short exact sequence of abelian groups for all R-modules D if and only if the original sequence is a split
short exact sequence.
Solution:
Dummit & Foote Text Exercise 10.5-16: This exercise proves Theorem 38 that every left R-module
M is contained in an injective left R-module.
(a) Show that M is contained in an injective Z-module Q. [M is a Z-module use Corollary 37.]
(b) Show that HomR (R, M ) HomZ (R, M ) HomZ (R, Q).
(c) Use the R-module isomorphism M
= HomR (R, M ) (Exercise 10) and the previous exercise to conclude
that M is contained in an injective module.
Solution:

MAT7400 Assignment #3

39

Dummit & Foote Text Exercise 10.5-19: If F is a field, prove that the injective hull of F is F .
Solution:
Dummit & Foote Text Exercise 10.5-22: Suppose that R is a commutative ring and that M and N
are flat R-modules. Prove that M R N is a flat R-module. [Use the previous exercise.]
Solution:
Dummit & Foote Text Exercise 10.5-26: Suppose R is a P.I.D. This exercise proves that A is a flat
R-module if and only if A is torsion free R-module (i.e., if a A is nonzero and r R, then ra = 0 implies
r = 0).
(a) Suppose that A is flat and for fixed r R consider the map r : R R defined by multiplication by
r : r (x) = rx. If r is nonzero show that r is an injection. Conclude from the flatness of A that the map
from A to A defined by mapping a to ra is injective and that A is torsion free.
(b) Suppose that A is torsion free. If I is a nonzero ideal of R, then I = rR for some nonzero r R. Show

that the map r in (a) induces an isomorphism R


= I of R modules and that the composite R I R of
r with the inclusion : I R is multiplication by r. Prove that the composite
1r
1
A R R A R I A R R corresponds to the map a 7 ra under the identification A R R = A and
that this composite is injective since A is torsion free. Show that 1 r is an isomorphism and deduce that
1 is injective. Use the previous exercise to conclude that A is flat.
Solution:
Dummit & Foote Text Exercise : .
Solution:
Dummit & Foote Text Exercise : .
Solution:
Dummit & Foote Text Exercise : .
Solution:

Das könnte Ihnen auch gefallen